Sei sulla pagina 1di 125

PROBLEMS ON TIME VALUE OF MONEY

1. Calculate the value of 5 years hence a deposit of Rs. 1,000 made today if the interest
rate is 8%, 10%, 12% and 15%.
Solution is as follows:
In the above question deposit is made only once for a period of 5 years. Hence, the
concept here is of future value single amount.
Future value = PV ( FVIFn=5 , i=8 % )
Future value = 1,000 x 1.469 = Rs. 1,469.00
Future value = 1,000 x 1.611 = Rs.1,611.00
Future value = 1,000 x 1.762 = Rs.1,762.00
Future value = 1,000 x 2.011 = Rs.2,011.00
2. Suppose you deposit Rs. 1,000 annually in a bank for 5 years and your deposits earn a
compound interest rate of 10 per cent. What will be the value of this series of deposits
at the end of 5 years? Assuming that each deposit occurs at the end of the year and
beginning of the year.
Solution is as follows:
Assuming deposits occurs at the end of the year
In the above question deposit is made annually at the end of the year for a period of 5
years. Hence, the concept here is of future value annuity.
Future value = PV ( FVIFA n= 4 , i=10% )
Future value = 1,000 x 4.641 = Rs.4,641.00
Assuming deposits occur at the beginning of the year
In the above question deposit is made annually at the beginning of the year for a
period of 5 years. Hence, the concept here is of future value annuity.
Future value = 1,000 x 6.105 = Rs.6,105.00

3. Fifteen annual payments of Rs. 5,000 are made into a deposit account that pays 14 per
cent interest per year. What is the future value of this annuity at the end of 15 years?
Solution is as follows:
In the above question deposit is made annually at the beginning of every year for a
period of 15 years. Hence, the concept here is of future value annuity.
Future value = PV ( FVIFA n=15 , i=14% )
Future value = 5,000 x 43.842 = Rs. 2,19,210.00

1|Page
4. If you invest Rs. 5,000 today at a compound interest of 9 per cent, what will be its future
value after 75 years?
Solution is as follows:
In the above question deposit is made only once for a period of 75 years. Hence, the
concept here is of future value single amount.
Future value = PV ( FVIFn=30 , i=9 % ) x PV ( FVIFn=30 , i=9 % ) xPV (FVIF n=15 , i=9 % )
Future value = 5,000 x (13.268) x (13.268) x (3.642)
Future value = Rs. 32,92,665.00

5. Mr. X wants to buy a house after 5 years when it is expected to cost Rs.2 million. How
much should he save annually if your savings earn a compound return of 12 per cent?
Solution is as follows:
In the above question, Mr.X wants to save some amount annually which earns 12%
interest and should yield to a future value of Rs. 2 million. We need to determine the
amount of deposit (present value) to be made annually. Hence, the concept is future
value annuity.
Future value
Pr esent value =
FVIFA n=5 ,i=12 %
Rs. 2 million
Pr esent value =
6.353
Pr esent value = Rs. 3,14,812.00

6. A finance company advertises that it will pay a lumpsum of Rs. 44,650 at the end of the
five years to investors who deposit annually Rs. 6,000 for 5 years. What is the interest
rate implicit in this offer?
Solution is as follows:
Interest rate can be calculated in two steps:
a. Firstly find the FVIFA value to know how many times the amount is growing
b. Secondly, by referring FVIFA table, find at what rate the FVIFA falls

Calculation of FVIFA
Future value 44 ,650
FVIFA = = = 7.442
Pr esent value 6,000
By looking at the FVIFA table – 4, 7.442 corresponding to 5th year row, falls exactly at
20%. So, we conclude that the interest rate offered here is 20%

2|Page
7. You can save Rs. 2,000 a year for 5 years, and Rs. 3,000 a year for 10 years thereafter.
What will these savings cumulate to at the end of 15 years, if the rate of interest is 10%.
Solution is as follows:
In the above question investor is saving Rs. 2,000 per year for first 5 years and there
on Rs.3,000 for next 10 years. It means that we need to determine the future value of
the savings. Hence, the concept here is future value annuity.
Future value = PV ( FVIFA n=15,i=10% ) + PV ( FVIFA n=5 ,i=10% )
Future value = 2,000 (31,772) + 1,000 (15.937)
Future value = 63,544 + 15,937 = Rs.79, 481

8. Mr. Vinay plans to send his son for higher studies abroad after 10 years. He expects the
cost of these studies to be Rs. 10,00,000. How much should he save annually to have a
sum of Rs. 10,00,000 at the end of 10 years, if the interest rate is 12 per cent.
Solution is as follows:
Mr. Vinay wants to have Rs. 10,00,000 in his account after 10 years. It means that he
should have a future value of Rs. 10,00,000 for that he should save some fixed amount
per year which carries interest of 12% per annum. Hence, the concept here is future
value annuity
Future value
PV =
FVIFA (n=10, i=12%)
¿
10,00,000
PV = = Rs. 56 ,983
17.549∗¿
*17.549 will appear in FVIFA (table – 4). We need to refer 10 th year row in 12%
column.

9. An Investor deposits Rs. 100 in a bank account for 5 years at 8% rate of interest. Find
out the amount which he will have in his account if interest is compounded annually,
semi-annually and quarterly.
Solution is as follows:
Future value if interest is compounded annually:
FV = PV x FVIF n=5 , i=8
FV = 100 x 1.469 = 146.90
Future value if interest is compounded semi-annually. In this case we need to double
the period and interest rate should be taken only 50%
FV = PV x FVIF n=10 , 1=4
FV = 100 x 1.480 = 148.00
Future value if interest is compounded quarterly. In this case we need to convert
period into 4 times and interest rate should be taken only 25%
FV = PV x FVIF n= 20 , i= 2%
FV = 100 x 1.486 = 148.60

3|Page
10. What is the present value of the following cash streams if the discount rate is 14%?
Year 0 1 2 3 4
Cash flow 5,000 6,000 8,00 9,000 8,000
0
Solution is as follows:
In the given solution we have future values. We need determine the present values of
these amounts. Hence, the concept here is present value single amount as every year
inflows are not equal.
Years Cash flows PVIFn=4 , i=14% Present value
(1) (2) (3) (4 = (2x3))
0 5,000 1.000 5,000
1 6,000 0.877 5,262
2 8,000 0.769 6,152
3 9,000 0.675 6,075
4 8,000 0.592 4,736
Total 27,225

11. Mahesh deposits Rs. 2,00,000 in a bank account which pays 10 per cent interest. How
much can he withdraw annually for a period of 15 years?
Solution is as follows:
In the given solution as on today Mahesh deposits Rs. 2,00,000 which is equal to
present value. Withdrawal amount per annum on his deposits will be on annuity basis.
The concept here is the total withdrawals for a period of 15 years is to be equal to the
present value of Rs. 2,00,000. Hence, the concept here is Present value annuity
Present value
FV =
PVIFAn=15, i=10
2,00 ,000
FV = = Rs.26,295
7.606

4|Page
12. Shyam borrows Rs.80,000 for a musical system at a monthly interest of 1.25 per cent.
The loan is to be repaid in 12 equal monthly instalments, payable at the end of each
month. Prepare the loan amortization schedule.
Solution is as follows:
The monthly installment of musical system is solved by using the following equation:
PV = EMI x PVIFA n=12, i=1 .25 %
1
1−
( 1 + r )n
PV = EMI x
r
1
1−
(1 + 0.0125 )12
80 ,000 = EMI x
0.0125
1
1−
(1.0125 )12
80 ,000 = EMI x
0.0125
80 ,000 = EMI x 11.0786
80 ,000
EMI = = Rs . 7,221
11.0786

The loan amortization schedule is shown below:


Loan amortization schedule
Mont Beginnin Monthly Interest Principa Balance at
h g amount installment repayment l the end
1 2 3 4 (2 x ROI) balance 6 (2-5)
5 (4-3)
1 80,000 7,221 1,000 6,221 73,779
2 73,779 7,221 922 6,299 67,480
3 67,480 7,221 844 6,377 61,103
4 61,103 7,221 764 6,457 54,646
5 54,646 7,221 683 6,538 48,108
6 48,108 7,221 601 6,620 41,488
7 41,488 7,221 519 6,702 34,786
8 34,786 7,221 435 6,786 28,000
9 28,000 7,221 350 6,871 21,129
10 21,129 7,221 264 6,957 14,172
11 14,172 7,221 177 7,044 7,128
12 7,128 7,221 89 7,132 (-4)*
*rounding off error

5|Page
13. Suppose a firm borrows Rs. 10,00,000 at an interest rate of 15% and the loan is to be
repaid in 5 equal installments payable at the end of the next 5 years. Prepare the loan
amortization schedule.
Solution is as follows:
In the given question firm has borrowed Rs. 10,00,000 in value which is equal to
Present value. He has to repay back the value to the borrower in equal annual
installments. Hence, the concept here is Present value annuity.
Pr esent value
Annual installment =
PVIFA n=5 , i=15%
10,00,000
Annual installment = = Rs. 2,98 ,329 .00
3.352
The loan amortization schedule is shown below:
Loan amortization schedule
Mont Beginnin Annual Interest Principa Balance at
h g amount installment repayment l the end
1 2 3 4 (2 x ROI) balance 6 (2-5)
5 (4-3)
1 10,00,000 2,98,329 1,50,000 1,48,329 8,51,671
2 8,51,671 2,98,329 1,27,751 1,70,578 6,81,093
3 6,81,093 2,98,329 1,02,164 1,96,165 4,84,928
4 4,84,928 2,98,329 72,739 2,25,590 2,59,338
5 2,59,338 2,98,329 38,901 2,59,428 90*
*due to rounding off an error, a small balance is shown

14. Mr. X’s father deposits Rs. 3,00,000 on retirement in a bank which pays 10 per cent
annual interest. How much can be withdrawn annually for a period of 10 years?
Solution is as follows:
Here in the given question Mr. X’s father is depositing Rs. 3,00,000 in bank which is
equal to present value. For a period of 10 years his son has to withdraw a value of Rs.
3,00,000 on equal amounts. Hence, the concept here is present value annuity.
Pr esent value
Amount =
PVIFA n=10, i=10%
3,00,000
Amount = = Rs. 48,820
6.145
Amount he can withdraw per annum is Rs. 48,820

6|Page
15. Mr. Suresh wants to save for the college education of his son, Deepak. He estimates that
the college education expenses will be rupees one million per year for four years when
his son reaches college in 16 years – the expenses will be payable at the beginning of the
year. He expects the annual interest rate of 8 per cent over the next two decades. How
much money should be deposit in the bank each year for the next 15 years (assume that
the deposit is made at the end of the year) to take care of his son’s college education
expenses?

Solution is as follows:
The time line for this problem is as follows:
0--------1---------2----------15----------16------------17------------18---------19
Now 1st 2nd 15th 1st 2nd 3rd 4th
Deposit Period Expenditure period
st
It is clear that deposits will begin from 1 year onwards and expenditure will begin from
16th year onwards. So for first 15 years it is of future value annuity and from 16 th
onwards it is present value annuity as it is expenditure period.
The present value of college education expenses when his son becomes 15 years old
is: Rs. 10,00,000 x PVIFA (n = 4, r = 8%)
Present value = 10,00,000 x 3.312 = Rs. 33,12,000

The annual deposit to be made so that the future value of the deposits at the end o 15
years is Rs. 33,12,000 is:
Future value
Amount =
FVIFA n=15 , r=8 %
33,12,000
Amount = = Rs. 1,21,980
27.152

16. What is the present value of Rs. 10,00,000 receivable 60 years from now, if the discount
rate is 10 per cent?

Solution is as follows:
In the given question future value is Rs. 10,00,000. We need to determine the present
value of the same amount. Hence, the concept here is present value single amount.
Pr esent value = Future value x ( PVIF (n=30 , i=10 %) )( PVIF ( n= 30 , i=10% ) )
Pr esent value = 10, 00 ,000 x 0 .057 x 0.057 = Rs . 3,249.00

7|Page
17. Calculate the following:
a. If you deposit Rs. 8,000 today at 8% rate of interest, in how many years will this
amount double, under rule 72 and 69?
Solution is as follows:
Doubling period under rule 72
72
Period =
Interest rate
72
Period = = 9 years
8

Doubling period under rule 69


69
Period = 0.35 +
Interest rate
69
Period = 0.35 + = 0.35 + 8.625 = 8.975 years
8
b. Mr. X will invest Rs. 5,000 at the end of each year for 6 years. Calculate its future
value if interest rate is 12% p.a.
Solution is as follows:
In the given question investment of Rs. 5,000 a year is made at the end of
every year. So investment will grow only for a period of 5 years. Hence, it is the
concept of future value annuity
Future value = Pr esent value x FVIFA n=5 , r=12%
Future value = 5,000 x 6.353 = Rs. 31,765
c. An employee is about to retire. His employer has offered him two post
retirement options: an annual pension of Rs. 10,000 as long as he lives and a
lumpsum payment of Rs. 60,000 if employee expects to live for 15 years and rate
of interest is 15%. Which alternative should be select?
Solution is as follows:
In the above question employer is offering two options:
i. Rs. 10,000 per year as long he lives
ii. Lumpsum of Rs. 60,000
Both the offers will have the concept of present valve. Option 1 is of
present value annuity and option 2 is of single amount.
Present value for option 1:
Pr esent value = FV x PVIFA n=15,r=15%
Pr esent value = 10,000 x 5.847 = Rs.58,470
Present value for option 2:
Employer is offering a lumpsum of Rs. 60,000 at the time of retirement. Rs.
60,000 is itself is of present value.
Among the two options, Present value of option 1 is more. So, he has to choose
option 1.

8|Page
18. Calculate the following:
a. Compute the future values of an investment of Rs.10,000 compounded semi-
annually for a period of 10 years at 10% p.a.
Solution is as follows:
FV = PV x FVIFn=20yrs, r=5%
FV = 10,000 x 2.488 = Rs. 24,880
b. A company has issued debentures of Rs. 50 lakhs to be repaid after 7 years. How
much the company should invest in a sinking fund earning 12% in order to be
able to repay debentures?
Solution is as follows:
Here the company has issued debentures for a period of 7 years which is of
present value. It has to repay the same amount in value after 7 years. When it
has to repay it has to save similar amount every year to avoid the burden of
one time saving. Hence, the concept here is future value annuity.
future value
Future value=
FVIFA n=7 , r=12%
50,00 ,000
Future value = = Rs. 4,95,589.25
10.089

c. A bank has offered to you an annuity of Rs. 1,800 for 10 years if you invest Rs.
12,000 today. What is the rate of return would you earn?
Solution is as follows:
The interest rate may be calculated in two steps:
Step 1: find the PVIFA for this contract by dividing PV by annuity inflows
present value
PVIFA n=10 years =
Annuity inf lows
12 ,000
PVIFA n=10 years = = 6.67
1,800
Step 2: look at the PVIFA table and read the row corresponding to 10 years
until you find a value close to 6.67. It falls in between 8% and 9%.
Approximately it is 8.5%

9|Page
19. Calculate the following:
a. ABC ltd., has Rs. 10 crore bonds outstanding. Bank deposit earns 10% p.a. the
bonds will be redeemed after 15 years for which purpose the company wishes to
create a sinking fund. How much amount should be deposited to the sinking
fund each year so that ABC ltd would have in the sinking fund 10 crore to retire
its entire issue of bonds.
Solution is as follows:
Here the company has issued debentures for a period of 15 years which is of
present value. It has to repay the same amount in value after 15 years. When it
has to repay it has to save similar amount every year to avoid the burden of
one time saving. Hence, the concept here is future value annuity.
future value
present value=
FVIFA n=15, r=10%
10,00, 00 ,000
present value = = Rs. 31, 47 ,425
31.772

b. Suppose someone offers you the following financial contract: if you deposit Rs.
10,000 with him he promises to pay Rs. 2,500 annually for 6 years. What interest
rate do you earn in this deposit?
Solution is as follows:
The interest rate may be calculated in two steps:
Step 1: find the PVIFA for this contract by dividing PV by annuity inflows
present value
PVIFA n=10 years =
Annuity inf lows
10 ,000
PVIFA n=10 years = = 4 .00
2,500
Step 2: look at the PVIFA table and read the row corresponding to 6 years until
you find a value close to 4.00. It falls 13%

20. Calculate the following:


a. Assume that a deposit is to be made at year zero into an account that will earn
8% compounded annually. It is desired to with draw Rs. 5,000 three years from
now and Rs. 7,000 six years from now. What is the size of year zero deposit that
will produce these future payments?
Solution is as follows:
From the above question it is known that we need to find the deposit amount
which is of present value (principal amount).
Amount = Inflows x PVIFA n=3 , r=8 %
Amount = 5,000 x 2.577 = Rs. 12,885
Amount = 7,000 x 5.206 = Rs. 36 ,442

10 | P a g e
b. Find out the present value of an investment which is expected to give a return of
Rs. 2,500 p.a. for 20 years and the rate of interest is 12% p.a.
Solution is as follows:
In the given question, inflows are of annuity. We need to determine the
present value of investment for future cash inflows. Hence, it is a concept of
present value annuity
PV of investment = Inflows x PVIFA n=20, r=12%
PV of investment = 2,500 x 7.469 = Rs. 18,672.50

21. Calculate the following:


a. XYZ ltd., has borrowed Rs. 5,00,000 to be repaid in five equal annual installments
(interest and principal). The interest rate is 16% p.a. compute the amount of
cash payment.
Solution is as follows:
From the above question we need to determine the value of annual payment
which is annuity. Hence, the concept here is present value annuity.
Present value
Installment amount =
PVIFA n=5 , r=16 %
5,00,000
Installment amount = = Rs. 1,52,718
3.274
b. The earnings of ABC ltd., was Rs 3 per share in year 1. It increased over a 10 year
period to Rs. 4.02. Compute the rate of growth or compound annual rate of
growth of the EPS.
Solution is as follows:
We need to compute the rate of interest earned for the increase in the
principal amount. The concept followed here is future value single amount
future value
Rate of int erest =
present value
4. 02
Rate of int erest = = 1. 34
3
FVIF of 1.340 has to be referred in table – 1 in 10 year row. It falls at 3%. Rate
of interest is 3%.

11 | P a g e
22. Calculate the following:
a. A borrower offers 16% nominal rate of interest with quarterly compounding.
What is the effective rate of interest?
Solution is as follows:
Interest is paid at the end of each quarter (m=4)
We need to calculate the annual percentage effective rate as follows:
AP /ER = ( 1 + r /m)m − 1.0
0.16 4
(
AP /ER = 1 +
4 ) − 1.0
4
AP /ER = ( 1 + 0.04 ) − 1.0
4
AP /ER = ( 1 .04 ) − 1.0
AP /ER = 1.1699 − 1.0 = 0.1699 or 16.99%

b. What is the present value of Rs. 10,00,000 receivable 30 years from now, if the
discount rate is 10%?
Solution is as follows:
In the above question future value is Rs. 10,00,000 receivable after 30 years
from now. We need to determine the present value of Rs. 30,00,000. Hence,
the concept here is present value single amount.
present value = Amount x PVIF n=30 , r=10%
present value = 10,00,000 x 0.073 = Rs. 73,000

c. Arun deposits Rs. 2,00,000 in a bank account which pays 10% interest. How
much can he withdraw annually for a period of 15 years?
Solution is as follows:
In the above question Arun has to withdraw equal amount every year. Hence,
the concept followed here is present value annuity
present value
Withdraw amount =
PVIFA n=15, r=10%
2 ,00,000
Withdrawel amount = = Rs. 26 ,295.00 per annum
7 .606

12 | P a g e
23. Calculate the following:
a. If you deposit Rs. 10,000 today at 10% rate if interest, in how many years will this
amount double?
Solution is as follows:
Here we need to apply doubling method under rule 72 or rule 69
72
Amount =
Interest rate
72
Amount = = 7.2 years
10
b. Compute the future value of an initial investment of Rs. 1,000 compounded
annually for 10 years at 10%, and
Solution is as follows:
Here we need to find the future value on initial investment (present value),
hence the concept followed here is future value single amount as deposit is
made only once for a period of 10 years.
Future value = Pr incipal x FVIFn=10, r=10
Future value = 1,000 x 2.594 = Rs. 2,594.00

c. An investor has two options to choose from the following:


i. Rs. 6,000 after I year, and
ii. Rs. 9,000 after 4 years.
Assume the discount rate of 20%, which alternative should he opt for?
Solution is as follows:
In the above question we need to determine the present value of both the
options, hence, the concept followed here is present value single amount.

Option 1
present value = Future value x PVIF n=1 , r=20%
present value = 6,000 x 0.833 = Rs. 5,000
Option 2
present value = Future value x PVIF n=4 , r=20%
present value = 9,000 x 0.482 = Rs. 4,338.00

Investor has to choose Option 1 as its present value is greater than Option 2.

13 | P a g e
24. Find out present values of the following:
a. Rs. 1,500 receivable in 7 years at a discount rate of 15%.
Solution is as follows:
present value = Amount x PVIFn=7 , r=15
present value = 1,500 x 0.376 = Rs. 564.00
b. An annuity of Rs.1000, starting immediately and lasting until 9 th year at a
discount rate of 20%.
Solution is as follows:
present value = Amount x PVIFAn=9 , r= 20
present value = 1,000 x 4.031 = Rs. 4 ,031.00
c. An annuity of Rs. 7600 starting after 1 year for 6 years at an interest rate of 12%.
Solution is as follows:
present value = Amount x PVIFA n=5 , r=12%
present value = 7,600 x 3.605 = Rs. 27 ,398.00
d. Operating expenditures of Rs. 1,00,000 per year which are assumed to be
incurred continuously for a period of 15 years, rate of interest is 12%.
Solution is as follows:
present value = Amount x PVIFA n=15 , r=12
present value = 1,00,000 x 6.811 = Rs. 6 ,81,100.00

25. Calculate the following:


a. A borrower offer 16% nominal rate of interest with quarterly compounding.
What is the effective rate of interest?
Solution is as follows:
Interest is paid at the end of each quarter (m=4)
We need to calculate the annual percentage effective rate as follows:
AP /ER = ( 1 + r /m)m − 1.0
0.16 4
(
AP /ER = 1 +
4 ) − 1.0
4
AP /ER = ( 1 + 0.04 ) − 1.0
4
AP /ER = ( 1 .04 ) − 1.0
AP /ER = 1.1699 − 1.0 = 0.1699 or 16.99%

b. A finance company advertises that it will pay a lump sum of Rs. 44,650 at the end
of five years to investors who deposit annually Rs. 6,000 for 5 years. What is the
interest rate implicit in this offer?
Solution is as follows:
Here interest rate is calculated in two steps:
Step 1: determine the FVIFA by dividing future value by deposits.

14 | P a g e
future value
FVIFA =
deposits
44,650
FVIFA = = 7.44
6,000
Step 2: refer FVIFA of 7.44 in table No – 2 corresponding row of 5 th year till you
find FVIFA of 7.44. It falls exactly at 20%. Hence, interest rate offered is 20%.
c. Mahesh deposits Rs. 2,00,000 in a bank account which pays 10 per cent interest.
How much can he withdraw annually for a period of 15 years.
Solution is as follows:
Here we need to determine the withdrawal which is of similar amount every
year. Hence, the concept followed here is present value annuity.
present value
withdrawal =
PVIFA n=15, r=10%
2,00,000
withdrawal = = Rs. 26,295.00
7,606
26. Calculate the following:
a. A bank finance a car purchase up to Rs. 2,50,000 at 12% interest per annum, for
a period of 5 years. How much equated monthly installments can be fixed in this
case?
Solution is as follows:
The monthly installment of musical system is solved by using the following
equation:
PV = EMI x PVIFA n=60, i=1 %
1
1−
( 1 + r )n
PV = EMI x
r
1
1−
( 1 + 0 .01 )60
2,50,000 = EMI x
0 .01
1
1−
( 1.01 )60
2,50,000 = EMI x
0 .01
2,50,000 = EMI x 44.955
2,50,000
EMI = = Rs. 5 ,561.12
44.955

15 | P a g e
b. An executive is about to retire at the age of 60. His employer has offered him
two post retirement options: (a) 2,50,000 per year for 10 years, (b) Rs. 20,00,000
lumpsum. Assuming 10% interest, which is a better option?
Solution is as follows:
In the above question employer is offering two options:
i. Rs. 2,50,000 per year as long he lives
ii. Lumpsum of Rs. 20,00,000
Both the offers will have the concept of present valve. Option 1 is of
present value annuity and option 2 is of present value single amount.
Present value for option 1:
Pr esent value = FV x PVIFA n=10 ,r=10%
Pr esent value = 2,50,000 x 6.145 = Rs. 15,35,250.00
Present value for option 2:
Employer is offering a lumpsum of Rs. 20,00,000 at the time of retirement. Rs.
20,00,000 is itself is of present value.
Among the two options, Present value of option 2 is more. So, he has to choose
option 2.

c. Mr. Raja requires Rs. 10 lakhs after 5 years. He considers the following 2 options:
i. To invest a single amount of 10% rate,
ii. To invest annually at a rate of 10% compounded annually. How much he
need to invest in both the cases?
Solution is as follows:
In option 1 the concept is future value single amount. We need to determine
the present value of single amount.
future value
Pr incipal =
FVIF n=5 , r=10%
10, 00,000
principal = = Rs. 6,20,732.00
1.611
In option 2 the concept is future value annuity. We need to determine the
present value of annuity.
future value
Pr incipal =
FVIFA n=5 , r=10 %
10,00,000
principal = = Rs. 1,63,800.00
6.105

16 | P a g e
27. An investor has an opportunity of receiving Rs. 1,000, Rs. 1,500, Rs. 800, Rs, 1,100 and
Rs. 400 respectively at the end of one through five years. Find out the present value of
this stream of uneven cash flows, if the investor’s required rate of return is 8 per cent.
Solution is as follows:
In the above question we have inflows, we need to determine the present values of
each inflows. Hence, the concept followed here is present value single amount.
Years Inflows PVIF PV inflows
1 1,000 0.926 926.00
2 1,500 0.857 1,285.50
3 800 0.794 635.20
4 1,100 0.735 808.50
5 400 0.681 272.40

28. If the discount rate is 10 per cent, compute the present value of the cash flow streams
detailed below: (a) Rs.100 at the end of year 1; (b) Rs. 100 at the end of year 4; (c) Rs.
100 at the end of year 3 and 5; (d) Rs. 100 for the next 10 years.
Solution is as follows:
(a) Present value of Rs. 100 at the end of year 1
PV = 100 x 0.909 = 90.90
(b) Present value of Rs. 100 at the end of year 4
PV = 100 x 0.683 = 68.30
(c) Present value of Rs. 100 at the end of year 3 and 5
PV (for the year ending 3) = 100 x 0.751 = 75.10
PV (for the year ending 5) = 100 x 0.621 = 62.10
(d) Present value of Rs. 100 at the end of year 10
PV = 100 x 0.386 = 38.60

29. ABC Ltd has borrowed Rs. 30,00,000 from Canbank Home Finance Ltd to finance the
purchase of a house for 15 years. The rate of interest on such loans is 24 per cent per
annum. Compute the amount of annual payment/installment.
Solution is as follows:
present value
Instalment =
PVIFA n=15, r=24%
30,00,000
Instalment = = Rs . 7,49, 812.55
4 .001
30. XYZ Ltd has borrowed Rs. 5,00,000 to be repaid in five equal annual payments (interest
and principal). The rate of interest is 6 per cent. Compute the amount of each payment.
Solution is as follows:
present value
Instalment =
PVIFA n=5 , r=6 %
5,00,000
Instalment = = Rs. 1,18,708
4 .212

17 | P a g e
SOLUTIONS SPECIFIC COST OF DEBT
1. A company has 10 per cent perpetual debt of Rs. 1,00,000. The tax rate is 35 per cent.
Determine the cost of capital (before tax as well as after tax) assuming the debt is issued
at par, 10 per cent discount and 10 per cent premium.
Solution is as follows:
Cost of debt is of non-redeemable
Cost of debt before tax
Issued at par Issued at 10% discount Issued at 10% premium
NP = FV = 100 NP = FV – D = 100 – 10 = 90 NP = Fv + P = 100 + 10 = 110
I I I
K db = x 100 K db = x 100 K db = x 100
NP NP NP
10 10 10
K db = x 100 = 10% K db = x 100 = 11.11% K db = x 100 = 9.09%
100 90 110
Cost of debt after tax
Issued at par Issued at 10% discount Issued at 10% premium
NP = FV = 100 NP = FV – D = 100 – 10 = 90 NP = Fv + P = 100 + 10 = 110
I ( 1−t ) I ( 1−t ) I ( 1−t )
K db = x 100 K db = x 100 K db = x 100
NP NP NP
10 ( 1−0.35 ) 10 ( 1−0 .35 ) 10 ( 1−0 .35 )
K db = x 100 = 6.5% K db = x 100 = 7. 22% K db = x 100 = 5 .91 %
100 90 110

18 | P a g e
2. Calculate the explicit cost of debt for each of the following situations:
a. Debentures are sold at par and floatation costs are 5 per cent.
b. Debentures are sold at premium of 10 per cent and floatation costs are 5 per
cent of issue price
c. Debentures are sold at discount of 5 per cent and floatation costs are 5 per cent
of issue price.
d. Assume coupon rate of interest on debentures is 10 per cent, face value of
debenture is Rs. 100 and maturity period is 10 years and tax rate is 35 per cent.
Cost of debt after tax
Redeemable debt (short cut metod)
Issued at par
I ( 1−t ) + ( f + d + pr + pi ) /n
K da = x 100
RV + NP/2
10 ( 1−0 .35 ) + ( 5 + 0 + 0 + 0 ) / 10
K da =
100 + 95 /2
10 ( 0. 65 ) + 0. 5
K da = x 100
97 .50
7
K da = x 100 = 7 .18 %
97 .50
Issued at premium
I ( 1−t ) + ( f + d + pr + pi ) /n
K da = x 100
RV + NP/2
10 ( 1−0.35 ) + ( 5 + 0 + 0 + 10 ) / 10
K da =
100 + 105/2
10 ( 0. 65 ) + 1. 5
K da = x 100
102.50
8
K da = x 100 = 7. 80 %
102.50
Issued at discount
I ( 1−t ) + ( f + d + pr + pi ) /n
K da = x 100
RV + NP/2
10 ( 1−0 .35 ) + ( 5 +5 + 0 + 0 ) / 10
K da =
100 + 90 /2
10 ( 0. 65 ) + 1
K da = x 100
95
7 .5
K da = x 100 = 7 .89 %
95

19 | P a g e
3. Abascus limited issued 15 year, 14 per cent bonds five years ago. The bond which has a
face value of Rs. 100 is currently selling for Rs. 108.00.
a. What is the pre tax cost of debt?
b. What is the after tax cost of debt? Assume 35 per cent tax rate.
Solution is as follows:
Cost of debt before tax Cost of debt after tax
Redeemable debt (short cut method) Redeemable debt (short cut method)

I + ( f + d + pr + pi ) /n I ( 1−t ) + ( f + d + pr + pi ) /n
K da = x 100 K da = x 100
RV + NP/2 RV + NP/2
14 + ( 0 + 0 + 0 + 8 ) / 15 14 ( 1−0 . 35 ) + ( 0 + 0 + 0 + 8 ) / 15
K da = K da =
100 + 108/2 100 + 108/2
14 + 0. 53 14 ( 0 . 65 ) + 0. 53
K da = x 100 K da = x 100
104 104
14 .53 9 .63
K da = x 100 = 13 . 97 % K da = x 100 = 9. 26 %
104 104

4. Assuming a corporate tax rate of 35 per cent, compute the after-tax cost of the capital in
the following situations:
a. A perpetual 15 per cent debentures of Rs. 1,000, sold at the premium of 10 per
cent with no floatation costs.
b. A ten year 14 per cent debentures of Rs. 2,000, redeemable at par, with 5 per
cent floatation costs.
Solution is as follows:

Cost of debt after tax Cost of debt after tax


Redeemable debt (short cut method) Redeemable debt (short cut method)

I ( 1−t ) + ( f + d + p r + pi ) /n I ( 1−t ) + ( f + d + p r + pi ) /n
K da = x 100 K da = x 100
RV + NP/2 RV + NP/2
150 ( 1−0 .35 ) + ( 0 + 0 + 0 + 100 ) / 10 280 ( 1−0. 35 ) + ( 100 + 0 + 0 + 0 ) / 10
K da = K da =
1000 + 1100/2 2000 + 1900/2
97.50 + 10 280 ( 0.65 ) + 10
K da = x 100 K da = x 100
1550 1950
107.50 192
K da = x 100 = 6.93% K da = x 100 = 9 .85 %
1550 1950

20 | P a g e
5. A company issues new 10 per cent debentures of Rs. 1,000 face value to be redeemed
after 10 years. The debenture is expected to be sold at 5 per cent discount. It will also
involve floatation costs of 5 per cent of face value. The company’s tax rate is 35 per
cent. What would the cost of debt be? Illustrate the computations using trial and error
method and short cut method.
Solution is as follows:
Cost of debt using Short cut method
I ( 1−t ) + ( f + d + pr + pi) /n
K da = x 100
RV + NP /2
10 (1−0.35) + ( 5 + 5 + 0 + 0 ) /10
K da = x 100
100 + 90 / 2
10 ( 0.65 ) + ( 1 )
K da = x 100
95
6.5 + 1
K da = x 100 = 7.89%
95
Cost of debt using Trial and Error Method
NP = I (1-t) (PVIFA10% , 10yrs ) + RV (PVIF10% , 10yrs )
90 = 10 (1-0.35) (6.145) + 100 (0.386)
90 = 10(0.65)(6.145) +38.60
90 = 39.94 + 38.60
90 = 78.54 (PV@HDF)

As redeemable value is less then NP, so, we need to increase its value by decreasing the
present value by adopting train and error method. Let us assume that New PVIF factor is 7%
NP = I (1-t) (PVIFA7% , 10yrs ) + RV (PVIF7% , 10yrs )
90 = 10 (1-0.35) (7.024) + 100 (0.508)
90 = 10 (0.65) (7.024) + 50.80
90 = 45.66 + 50.80
90 = 96.46 (PV@LDF)

At 10% PVIF is 78.54 and at 7% it is 96.46. It implies that NP falls in between 10% and
7%. By applying IRR equation, we need to determine cost of debt assuming 7% as LDF
and 10% as HDF.
PV @ LDF − II
K da = LDF + [PV @ LDF − PV @ HDF ]
x [ HDF − LDF ]

96 .46 − 90
K da = 7 + [
96 .46 − 78 .54 ]
x [ 10 −7 ]

6 .46
K da = 7 + [ ]
17.92
x3
K da = 7 + 1 . 08 = 8 .08 %

21 | P a g e
6. Calculate the explicit cost of debt for each of the following situations:
a. Debentures are sold at par and floatation costs are 5 per cent.
b. Debentures are sold at premium of 10 per cent and floatation costs are 5 per
cent of issue price
c. Debentures are to repaid in 10 equal annual installments commencing from year
end 1.
Solution is as follows:
If debentures are issued at par with floatation cost of 5% = NP (II)= 95
PVIF PV of cash flows
Year Outflows
7% 8% 7% 8%
1 *16.50 0.935 0.926 15.43 15.28
2 **15.85 0.873 0.857 13.84 13.58
3 15.20 0.816 0.794 12.40 12.07
4 14.55 0.763 0.735 11.10 10.69
5 13.90 0.713 0.681 9.91 9.47
6 13.25 0.666 0.630 8.82 8.35
7 12.60 0.623 0.583 7.85 7.35
8 11.95 0.582 0.540 6.95 6.45
9 11.30 0.544 0.500 6.15 5.65
10 10.65 0.508 0.463 5.41 4.93
97.86 93.82
Working note: calculation of outflows
Installment amount Rs. 10 (100/10)
For year 1 = Rs. 10 + interest at 10% on Rs.100 (1-t)
*For year 1 = Rs. 10 + Rs. 10 (1-0.35) = 16.50
For year 2 = Rs. 10 + interest at 10% on Rs. 90 (1-t)
** for year 2 = Rs. 10 + Rs. 9 (1-0.35) = 15.85
PV @ LDF − II
K da = LDF + [ PV @ LDF − PV @ HDF]x [ HDF − LDF ]

97 .86 − 95
[
K da = 7 + ]
97 .86 − 93.82
x [8 − 7]

2. 86
[ ]
K da = 7 +
4 .04
x1
K da = 7 + 0 . 71 = 7.71 %

22 | P a g e
If debentures are issued at premium of 10% and floatation cost of 5% - II = 105
PVIF PV of cash flows
Year Outflows
5% 6% 5% 6%
1 *16.50 0.952 0.943 15.71 15.56
2 **15.85 0.907 0.890 14.38 14.11
3 15.20 0.864 0.840 13.13 12.77
4 14.55 0.823 0.792 11.97 11.52
5 13.90 0.784 0.747 10.89 10.38
6 13.25 0.746 0.705 9.88 9.34
7 12.60 0.711 0.665 8.38 8.38
8 11.95 0.677 0.627 8.09 7.49
9 11.30 0.645 0.592 7.29 6.69
10 10.65 0.614 0.558 6.54 5.94
106.26 102.18
Working note: calculation of outflows
Installment amount Rs. 10 (100/10)
For year 1 = Rs. 10 + interest at 10% on Rs.100 (1-t)
*For year 1 = Rs. 10 + Rs. 10 (1-0.35) = 16.50
For year 2 = Rs. 10 + interest at 10% on Rs. 90 (1-t)
** for year 2 = Rs. 10 + Rs. 9 (1-0.35) = 15.85
PV @ LDF − II
K da = LDF + [ PV @ LDF − PV @ HDF ]x [ HDF − LDF ]

106 .26 − 105


K da = 5 +[ 97 .86 − 93.82] x [ 6 − 5]

1 . 26
K da = 5 +[ ]
4 .08
x1
K da = 5 + 0 . 43 = 5 . 43 %

23 | P a g e
7. A company issued 11 per cent debentures of Rs. 100 for an amount aggregating Rs.
1,00,000 at 10 per cent premium, redeemable at par after 5 years. The company’s tax
rate is 35 per cent. Determine the cost of debt using trial and error and short cut
method.
Solution is as follows:
Cost of debt using Short cut method
I ( 1−t ) + ( f + d + pr − pi ) /n
K da = x 100
RV + NP /2
11 (1−0.35) + ( 0 + 0 + 0 − 10 ) /5
K da = x 100
100 + 110 / 2
11 ( 0.65 ) + (−2 )
K da = x 100
105
7.15 − 2
K da = x 100 = 4.90%
105
Cost of debt using Trial and Error Method
NP = I (1-t) (PVIFA11% , 5yrs ) + RV (PVIF11% , 5yrs )
110 = 11 (1-0.35) (3.696) + 100 (0.593)
110 = 11(0.65)(3.696) +59.30
110 = 26.43 + 59.30
110 = 85.73 (PV@HDF)

As redeemable value is more than NP, so, we need to increase its value by decreasing the
present value by adopting train and error method. Let us assume that New PVIF factor is 10%
NP = I (1-t) (PVIFA7% , 5yrs ) + RV (PVIF7% , 5yrs )
110 = 11 (1-0.35) (5.786) + 100 (0.713)
110 = 11 (0.65) (5.786) + 71.30
110 = 41.37 + 71.30
110 = 112.67 (PV@LDF)

At 11% PVIF is 85.73 and at 7% it is 112.67. It implies that NP falls in between 11% and
7%. By applying IRR equation, we need to determine cost of debt assuming 7% as LDF
and 10% as HDF.
PV @ LDF − II
K da = LDF + [PV @ LDF − PV @ HDF ]
x [ HDF − LDF ]

112. 67 − 110
K da = 7 + [
112. 67 − 85 .73 ]
x [ 11 − 7 ]

2. 67
K da = 7 + [ ]
26 .94
x4
K da = 7 + 0 . 40 = 7 . 40 %

24 | P a g e
8. Information is as follows:
a. A company’s debentures of the face value of Rs. 100 bear as 8 per cent coupon
rate. Debentures of this type currently yield 10 per cent. What is the market
price of debentures of the company?
b. What would happen to the market price of the debentures if interest rises to 16
per cent and drops to 12 per cent?
c. What would be the market price of the debentures in situation (a) if it is
assumed that debentures were originally having a 15 maturity period is 4 years
away from now?
d. Would you pay Rs.90 to purchase debentures specified in situation (c)? Explain.
Solution is as follows:
Interest
Market value =
Rate of current yield
8
Market value = = Rs . 80 .00
10 %
Market value if interest rises to 16% and drops to 12%
8
Market value = = Rs.50.00
16%
8
Market value = = Rs . 66.67
12%
Market value of debentures if it is matured after 4 years (15-11)
MV = I (PVIFA4yrs, 10%) + RV (PVIF4years, 10%)
MV = 8 (3.170) + 100 (0.683)
MV = 25.36 + 68.30
MV = Rs. 93.66

Yes, we would pay Rs. 90 for the purchase of debentures because it’s current
worth (93.66) is more than the purchase price.

9. ABC company sold Rs. 1,000 6 per cent debentures carrying on maturity date to the
public a decade earlier. Interest rates since have risen so that debentures of the quality
represented by this company are now selling at 9 per cent yield basis:
a. Determine the current indicated market price of the debentures. Would you pay
the debenture for Rs. 700? Explain your answer
Solution is as follows:
Interest
Market value =
Rate of current yield
60
Market value = = Rs . 667 . 00
9%

We would not pay Rs. 700 for the purchase of debenture as its current yield (667)
is less than the purchase price.

25 | P a g e
10. The Elu Ltd is contemplating a debenture issue on the following terms:
a. Face value Rs. 100 per debenture.
b. Term of maturity 7 years
c. Yearly coupon rate of interest:
i. Years 1 – 2 : 9 per cent
ii. Years 3 – 4 : 10 per cent
iii. Years 5 – 7 : 11 per cent
d. The current market rate on similar debentures is 11 per cent per annum. The
company proposes to price the issue so as to yield a return on 12 per cent per
annum to the investors. Determine the issue price. Assume redemption at a
premium of 5 per cent of face value.
Solution is as follows:
In the above question rate of interest in changing every year. So, interest payment
amount will be of single amount concept.
Years Interest principal Outflow PVIF@ PV of cash
(I + p) 12% outflows
1 9.00 9.00 0.893 8.037
2 9.00 9.00 0.797 7.173
3 10.00 10.00 0.712 7.120
4 10.00 10.00 0.636 6.360
5 11.00 11.00 0.567 6.237
6 11.00 11.00 0.507 5.577
7 11.00 105 116.00 0.452 52.432
Issue Price 92.936

26 | P a g e
11. A company is considering raising Rs. 100 lakhs by one of the two alternative methods,
viz., 14 per cent institutional term loan and 13 per cent non-convertible debentures. The
term loan option would attract no major incidental cost. The debentures would have to
be issued at a discount of 2.5 per cent and would involve Rs. 1 lakh as cost of issue.
Advise the company as to the better option on the effective cost of
capital in each case. Assume a tax rate of 35 per cent.

Solution is as follows:
a. Cost of debt by rising institutional term loan
int erest
K da = x 100
Net proceeds
14,00,000(1−0.35)
K da = = 9.10%
1,00,00,000
b. Cost of debt by rising non-convertible debentures
interest
Kda = x 100
Net proceeds
13,00,000(1−0.35)
Kda = =8.76%¿∗96,50,000=1,00,00,000−2,50,000−1,00,000
96,50,000∗¿
Rising of funds through non-convertible debentures is a better option.

27 | P a g e
SOLUTION AND PROBLEMS ON RETAINED EARNINGS AND EQUITY CAPITAL
1. A company declared a dividend of Rs. 2 per share, market price per share is Rs. 20,
income tax rate is 60 per cent and expected brokerage is to be 2 per cent. Compute the
cost of retained earnings
Solution is as follows:
1− t
K re = K e [ ]
1 − br
D 2
Ke = x 100 = x 100 = 10%
CMP or NP 20
1 − 0.60
K re = 10 [ ]
1 − 0.02
0.40
K re = 10
[ ]
0.98
K re = 10 [ 0.408 ]
K re = 4.08%

2. ABC Company’s cost of equity is 14 per cent, the average tax rate of an individual
shareholder is 40 per cent and it is expected to spend 2 per cent on brokerage cost that
shareholders will have to pay while investing their dividends in alternative securities.
What is the cost of retained earnings?
Solution is as follows:
1− t
K re = K e [ ]
1 − br
1 − 0.40
K re = 14 [ ]
1 − 0.02
0.60
K re = 14 [ ]
0.98
K re = 14 [ 0.612 ]
K re = 8.57%

28 | P a g e
3. Life style garment manufacturing company has a net earnings of Rs, 20,00,000 and all of
its shareholders are in the tax bracket of 50 per cent. The management estimates that
under present conditions stockholder’s required rate of return is 10 per cent. 3 per cent
is the expected brokerage to be paid if stockholders want to invest in alternative
securities. Compute the cost of retained earnings.
Solution is as follows:
1− t
K re = K e [ ]
1 − br
1 − 0.50
K re = 10 [ ]
1 − 0.03
0.50
K re = 10 [ ]
0.97
K re = 10 [ 0.515 ]
K re = 5.15%

4. BPL company’s equity share is currently selling at Rs. 350.75 and it is currently paying a
dividend of Rs. 5.25 per share. The dividend is expected to grow at a 15 per cent per
annum for one year. Income tax rate is 40 per cent and brokerage is 2 per cent.
Calculate cost of retained earnings.
Solution is as follows:
1− t
K re = K e
[ ]
1 − br
1 − 0.40
K re = 16.50∗ [ ]
1 − 0.02
0.60
K re = 16.50 [ ]
0.98
K re = 16.50 [ 0.612 ]
K re = 10.09%
D
¿ Ke = x 100 + G
CMP
5.25
¿ Ke = x 100 + 15 = 16.50
350.75

29 | P a g e
5. Company has earnings available to ordinary shareholders Rs. 5,00,000. It has equity
capital of Rs. 50,00,000 face value of Rs. 100 each. The company’s share is selling at Rs.
200. Compute the cost of equity (assuming 100 per cent dividend payout ratio).
Solution is as follows:
Cost of equity based on face value:
D
Ke = x 100
CMP
10∗¿
Ke = x 100 = 10% ¿Cost of equity based on market value :
100
D
Ke = x 100
CMP
10∗¿ Dividend paid
Ke = x 100 = 5% ¿∗DPS =
200 No.of equity shares
5,00,000
DPS = = Rs. 10 per share
50,000**
Equity capital 50,00,000
**No of equity shares = = = 50,000 shares
Face value of share 100
6. Company is currently earning Rs. 10,00,000 and its share is selling at a market price of
Rs. 160. The firm has 2,00,000 shares outstanding and has no debt. The earnings of the
firm are expected to remain stable and it has a payout ratio of 100 per cent. What is K e?
If firm’s payout ratio is assumed to be 70 per cent and that it earns 15 per cent of return
on its investment opportunities, then what would be the firms Ke.
Solution is as follows:
Cost of equity based on 100% payout:
D
Ke = x 100
CMP
5∗¿ 10,00,000
K e = x 100 = 3.125% ¿∗D = = Rs. 5 per share
160 2,00,000
Cost of equity based on 70% payout:
D
Ke = x 100 + G
CMP
3.50
Ke = x 100 + 4.50∗= 6.688%
160
¿G = (Retention X rate of returns)= 30 x 15% = 4.50

30 | P a g e
7. The shares of a company are selling at Rs. 20 per share. The firm had paid dividend at
Rs. 2 per share last year. The estimated growth of the company is approximately 5 per
cent per year.
a. Determine the cost of equity capital of the company.
b. Determine the estimated market price of the equity shares if the anticipated
growth rate of the firm rises to 8% and falls to 3%.
Solution is as follows:
Cost of equity if growth rate is 5 %
D
Ke = x 100
CMP
2.10
Ke = x 100 + 5 = 15.5 %
20
Market price of equity if growth rate rises to 8 %
D
Po =
[Ke − g]
2 .16
Po = = Rs . 28. 80
[ 0 .155 − 0.08 ]
Market price of equity if growth rate falls to 3 %
D
Po =
[ K e + g]
1 .94
Po = = Rs . 10. 49
[ 0 .155 + 0. 03 ]
8. Equity share of a paper manufacturing company is currently selling at Rs. 100. It wants
to finance its capital expenditure of Rs. 1,00,000 either by retaining earnings or selling
new shares. If company seeks to sell shares, the issue price will be Rs. 95. The expected
dividend for the next year is Rs. 4.75 and it is expected to grow at 6% perpetually.
Calculate the cost of equity capital.
Solution is as follows:
Cost of equity based on face value:
D
K e = x100 +g
CMP
75∗¿
K e =4. x 100+6=10.75%¿Cost of equity based on market value:
100
D
K e = x100
CMP
76∗¿
K e =4. x 100+6=11%¿
95

31 | P a g e
9. From the following dividends record of a company compute the expected growth rate:
Year 1 2 3 4 5 6 7 8
Dividend per 2 22 2 2 25 2 2 28
share 1 3 4 6 7

Solution is as follows:
In the above given question we need to determine the growth rate by taking dividend
of 1st and 8th year:
Dn
gr: (1 + r )n =
Do
n 28
gr: (1 + r ) = = 1 .333
21
By referring future value sin gle amount ( FVIF) for 7 years , 1. 333 exactly falls at 4 %.
Growth rate = 4 %

10. Mr. A an investor purchases an equity share of a growing company for Rs. 210. He
expects the company to pay dividends of Rs. 10.50, Rs. 11,025 and Rs. 11.575 in years 1,
2 and 3 respectively. He expects to sell the shares at a price of Rs. 243. 10 at the end of
three years. Determine dividend growth rate, current dividend yield and required rate of
return on Mr. A’s equity investment.
Solution is as follows:
Growth rate:
Dn
gr: (1 + r )n =
Do
11. 575
gr: (1 + r )n = = 1.102
10.50
By referring future value sin gle amount ( FVIF) for 2 years , 1. 102 exactly falls at 5 %.
Growth rate = 5%

Current dividend yield: in simple, current dividend yield is equal to growth rate of
dividends.

Mr. A’s required rate of return:


Dividend per share
Ke = x 100 + G
expected sales price
11.575 + (5% of 11.575)
Ke = x 100 + 5= 10%
243.10

32 | P a g e
11. The capital ltd., wishes to calculate its cost of equity capital using the CAPM model.
Company’s analyst found that its risk free rate of return equals 12%, beta equals 1.7 and
the return on market portfolio equals 14.5%.
Solution is as follows:
Ke = Rf + β (Rm −Rf )
Ke = 0.12 + 1.70 (0.145 − 0.12)
Ke = 0.12 + 1.70 (0.025)
Ke = 0.12 + 0.0425 = 0.1625 or 16.25%

12. Calculate the required rate of return on four equity stocks with the beta values shown
against them:
Stoc A B C D
k
Beta 0.90 1.1 1.60 1.90
0
The risk free rate is 20% and rate of return on the market portfolio is 32%.
Solution is as follows:
For Stock A:
Ke = Rf + β (Rm −Rf )
Ke = 0.20 + 0.90 (0.32 − 0.20)
Ke = 0.20 + 0.90 (0.12)
Ke = 0.20 + 0.108 = 0.308or 30.80%

For Stock B:
Ke = Rf + β (Rm −Rf )
Ke = 0.20 + 1.10 (0.32 − 0.20)
Ke = 0.20 + 1.10 (0.12)
Ke = 0.20 + 0.132 = 0.332or 33.20%

For Stock C:
Ke = Rf + β (Rm −Rf )
Ke = 0.20 + 1.60 ( 0.32 − 0.20)
Ke = 0.20 + 1.60 ( 0.12)
Ke = 0.20 + 0.192 = 0.392or 39.20%

For Stock D:
Ke = Rf + β (Rm −Rf )
Ke = 0.20 + 1.90 (0.32 − 0.20)
Ke = 0.20 + 1.90 (0.12)
Ke = 0.20 + 0.228 = 0.428or 42.80%
33 | P a g e
13. An investor supplied you the following information and requested to you to calculate:
a. Expected rate of return of market portfolio
b. Expected returns on each security, using CAPM
Year-end Beta risk
Investment in company Initial price Dividends
market price factor
Paper 20 2 55 0.70
A Steel 30 2 65 0.80
Chemical 40 2 140 0.60
B GOI Bonds 1,000 140 1,005 0.99
Risk free returns are 10 per cent.

Solution is as follows:
1. Calculation of expected rate of return on market portfolio:
Security Returns
Dividend Capital gain Total Investments
A Paper 2 35 (55-20) 37 20
Steel 2 35 (65-30) 37 30
chemical 2 100 (140-40) 102 40
B GOI Bonds 140 5 (1005-1000) 145 1000
Total 321 1090

Total return
Market return = x 100
Total Investment
321
Market return = x 100 = 29 . 45 %
1.090
2. Cost of equity using CAPM approach
For paper company
Ke = Rf + β (Rm −Rf )
Ke = 0.10 + 0.70 ( 0.2945 − 0.10 ) = 0.10 + 0.1362 = 0.2362 or 23.62%
For steel company
Ke = Rf + β (Rm −Rf )
Ke = 0.10 + 0.80 ( 0.2945 − 0.10 ) = 0.10 + 0.1556 = 0.2556 or 25.56%
For chemical company
Ke = Rf + β (Rm −Rf )
Ke = 0.10 + 0.60 ( 0.2945 − 0.10 ) = 0.10 + 0.1167 = 0.2167 or 21.67 %
For GOI Bonds
Ke = Rf + β (Rm −Rf )
Ke = 0.10 + 0.99 ( 0.2945 − 0.10 ) = 0.10 + 0. 193= 0.293 or 29.30%

34 | P a g e
14. An investor supplied you the following information and requested to you to calculate:
a. Expected rate of return of market portfolio
b. Expected returns on each security, using CAPM
Year-end Beta risk
Investment in company Initial price Dividends
market price factor
X 50 5 95 0.9
A Y 71 13 125 0.7
Z 81 16 144 1.6
B GOI Bonds 705 52 800 1.7
Risk free returns are 12 per cent.

Solution is as follows:
1. Calculation of expected rate of return on market portfolio:
Security Returns
Dividend Capital gain Total Investments
A Paper 5 45 50 50
Steel 13 54 67 71
chemical 16 63 79 81
B GOI Bonds 52 95 147 705
Total 343 907

Total return
Market return = x 100
Total Investment
343
Market return = x 100 = 37 . 82 %
907
2. Cost of equity using CAPM approach
For paper company
Ke = Rf + β (Rm −Rf )
Ke = 0.12 + 0.90 ( 0.3782 − 0.12 ) = 0.12 + 0.2324 = 0.3524 or 35.24%
For steel company
Ke = Rf + β (Rm −Rf )
Ke = 0.12 + 0.70 ( 0.3782 − 0.12 ) = 0.12 + 0.1807 = 0.3007 or 30.07%
For chemical company
Ke = Rf + β (Rm −Rf )
Ke = 0.12 + 1.60 ( 0.3782 − 0.12 ) = 0.12 + 0.4131 = 0.5331 or 53.31%
For GOI Bonds
Ke = Rf + β (Rm −Rf )
Ke = 0.12 + 1.70 ( 0.3782 − 0.12 ) = 0 .12 + 0. 4389= 0.5589 or 55.89%

35 | P a g e
15. Om Sai Enterprises issued 9 per cent preference share (irredeemable) four years ago.
The preference share that has a face value of Rs.100 is currently selling at Rs. 93. What
is the cost of preference share with 8 per cent dividend tax?

Solution is as follows:
PD (1+t )
K pda = x 100
NP
9 (1−0.08)
K pda = x 100 = 8.90%
93

16. A company is considering the most desirable capital structure, the following estimates
of the debt and equity capital (after tax) have been made at various levels of debt-
equity mix.
Debt as a % of Cost of Cost of
capital debt equity
0 6.0 13
10 6.0 13
20 6.0 13.5
30 6.5 14
40 7.0 15
50 7.5 17
Determine the optimal debt-equity mix for the company by calculating the overall cost
of capital

Solution is as follows:
Debt as a % Equity as a % Cost of Cost of Overall cost of capital
of capital of capital debt equity [ W d xK d ] + [ W e x K e ]
W
[ d] W
[ e] K
[ d] K
[ e]
0 100 6.0 13 [0x.06] + [1x.13] = .13 or 13%
10 90 6.0 13 [.1x.06]+[.9x.13] = .123 or 12.3%
20 80 6.0 13.5 [.2x.06]+[.8x.135] = .12 or 12%
30 70 6.5 14 [.3x.065]+[.7x.14] = .1175 or 11.75%
40 60 7.0 15 [.4x.07]+[.6x.15] = .118 or 11.8%
50 50 7.5 17 [.5x.075]+[.5x.17] = .1225 or 12.25%

30 per cent debt and 70 per cent equity gives the optimal debt-equity mix or optimal
capital structure, since at that proportion of debt-equity mix the overall cost of capital
is minimum.

36 | P a g e
17. AB Ltd estimates the cost of equity and debt components of its capital for different
levels of debt : equity mix as follows:
Debt as a % of Cost of Cost of
capital debt equity
0 12 16
20 12 16
40 16 20
Suggest the best debt: equity mix for the company. Tax rate is 50 per cent.
Solution is as follows:
Debt as a % Equity as a % Cost of Cost of Overall cost of capital
of capital of capital debt equity [ W d xK d ] + [ W e x K e ]
[Wd ] [W e ] K d [ 1−t ] [ K e]
0 100 6.0 16 [0x.06] + [1x.16] = .16 or 16%
20 80 6.0 16 [.2x.06] + [.8x.16] = .14 or 14%
40 60 8.0 20 [.4x.08] + [.6x.20] = .152 o 15.2%

20 per cent debt and 80 per cent equity gives the optimal debt-equity mix or optimal
capital structure, since at that proportion of debt-equity mix the overall cost of capital
is minimum.

18. A firm finances all its investments by 40 per cent debt and 60 per cent equity. The
estimated required rate of return on equity is 20 per cent after-taxes and that of the
debt is 8 per cent after taxes. The firm is considering an investment proposal costing Rs.
40,000 with an expected return that will last forever. What amount (in rupees) must the
proposal yield per year so that the market price of the share does not change? Show
calculations to prove your point.

Solution is as follows:
The firm has to find the weighted average cost of capital:
WACC = [ W d xK d ] + [ W e x K e ]
WACC = [ 0.40 x 20 ] + [ 0.60 x 8 ]
WACC = 8 + 4.8 = 12.80%
The company has to yield a minimum of Rs. 5,120 (40,000 x 12.80%) per year so that
the market price of the share does not change.

19. A company has 1,00,000 shares of Rs. 100 at par, of preference shares, outstanding at
9.75 per cent dividend rate. The current market price of the preference share is Rs. 80.
What is its cost?
Solution is as follows:

37 | P a g e
PD 9.75
K pd = x 100 = K pd = x 100 = 12.188%
NP or CMP 80

20. A company has on its books the following amounts and specific cost of each type of
capital:
Sources of capital Book value Market value Specific
costs (%)
Debt 4,00,000 3,80,000 5
Preference 1,00,000 1,10,000 8
EQUITY 12,00,000
Equity share capital 6,00,000 (equity) 15
Retained earnings 2,00,000 13
Determine the weighted average cost of capital of the company on book value and
market value approaches.

Solution is as follows:
Weighted average cost of capital on book value weights:
Sources of capital Book value Book value (in K WACC
weights) (W) (after tax) (W x K)
Debt 4,00,000 0.31 5 1.55
Preference 1,00,000 0.08 8 0.64
Equity share capital 6,00,000 0.46 15 6.90
Retained earnings 2,00,000 0.15 13 1.95
13,00,000 1.00 11.04%

Weighted average cost of capital on market value weights:


Sources of capital Market Market value K WACC
value (in weights)(W) (after tax) (W x K)
Debt 3,80,000 0.22 5 1.10
Preference 1,10,000 0.07 8 0.56
Equity share capital* 9,00,000 0.53 15 7.95
Retained earnings** 3,00,000 0.18 13 2.34
16,90,000 1.00 11.95%

Working note:
EQUITY = EQUITY SHARE CAPITAL + RETAINED EARNINGS
*calculation of equity share capital and retained earnings of market value.
In book value weights, equity and retained earnings are in the ratio of 6:2 or 3:1. In
the same ratio, we need to divide for market value:

38 | P a g e
12,00 ,000 x 3
¿ Equity = = 9,00,000
4
12,00,000 x 1
**Retained earnings = = 3,00,000
4,00,000

21. Aries limited wishes to raise additional finance of Rs. 10 lakhs for meeting its investment
plans. It has Rs. 2,10,000 in the form of retained earnings available for investment
purposes. The following are the further details:
a. Debt-equity mix: 30:70
b. Cost of debt: upto Rs. 1,80,000, 10 per cent (before tax); beyond Rs. 1,80,000, 12
per cent (before tax).
c. Earnings per share: R.4
d. Dividend payout, 50 per cent of earnings
e. Expected growth rate in dividend, 10 per cent
f. Current market price per share Rs. 44
g. Tax rate 35 per cent.
You are required:
 To determine the pattern for raising the additional finance, assuming the firm
intends to maintain existing debt/equity mix.
 To determine the post-tax average cost of additional debt.
 To determine the cost of retained earnings and cost of equity
 Compute the overall weighted average after tax cost of additional finance.

Solution is as follows:
1. Pattern for raising additional finance:
Debt - equity mix = 30:70
Additional finance – Rs. 10,00,000
Debt = 10,00,000 x 0.30 = 3,00,000
Equity = 10,00,000 x 0.70 = 7,00,000
Sources of capital Amount Total
Equity:
 Equity share capital 4,90,000
 Retained earnings 2,10,000 7,00,000
Debt:
 10% Debt 1,80,000
 12% Debt 1,20,000 3,00,000
Total funds 10,00,000

2. Post-tax average cost additional debt:

39 | P a g e
Interest
K da = x 100
Total Debt capital
[10% of 1,80,000 ] + [12% of 1,20,000 ] [ 1−t ]
K da = x 100
3,00 ,000
18,000 + 14,400 [ 1−0.35 ] 32,400 [ 0.65 ] 21,060
K da = x 100 = x 100 = x 100 = 7.02%
3,00 ,000 3,00,000 3,00,000

3. Cost of retained earnings and cost of equity:


Dt
Ke = +g
Po
4 ( 50% ) + ( 10% )
Ke = + 10%
44
2 + 10 % of 2 2 + 0.2 2.2
Ke = + 10% = + 10% = + 10 % = 15 %
44 44 44
K e = 15%
K r = K e = 15%
4. Overall cost of capital of additional finance:
Sources of capital Market Market value K WACC
value (in weights)(W) (after tax) (W x K)
Equity share capital 4,90,000 0.49 15.00 7.35
Retained earnings 2,10,000 0.21 15.00 3.15
Debt 3,00,000 0.30 7.02 2.106

10,00,000 1.00 12.606%

22. As a financial analyst of a large electronics company, you are required to determine the
weighted average cost of capital of the company using (a) book value weights and (b)
market value weights. The following information is available for your perusal:
The company’s present book value capital structure is:
Debentures (Rs. 100 per debenture) Rs. 8,00,000
Preference shares (Rs. 100 per share) Rs. 2,00,000
Equity shares (Rs. 10 per share) Rs. 10,00,000
Total capital Rs. 20,00,000
All these securities are traded in the capital markets. Recent prices are: debentures Rs.
110 per debenture, Preference shares Rs. 120 per share and equity shares Rs. 22 per
share.
Anticipated external financing opportunities are:
 Rs. 100 per debenture redeemable at par; 10 year maturity, 11 per cent coupon
rate, 4 per cent floatation cost, sale price Rs. 100.

40 | P a g e
 Rs. 100 preference shares redeemable at par; 10 year maturity, 12 per cent
dividend rate, 5 per cent floatation costs, sale price Rs. 100
 Equity shares: Rs. 2 per share floatation costs, sale price Rs. 22.
In addition, the dividend expected on the equity share at the end of the year is
Rs. 2 per share; the anticipated growth rate in dividends is 7 per cent and the
firm has the practice of paying all its earnings in the form of dividends. The
corporate tax rate is 35 per cent.

Solution is as follows:
Initially we need to determine the specific cost of all the sources of capitals:
 Cost of debt capital (redeemable under short cut method):
I ( 1−t ) + ( f +d+ pr+ pi) /n
K da = x 100
( RV + NP ) /2
11 ( 1−0 .35 ) + ( 4+0+0+0 ) /10 7 .15 + 0. 4
K da = x 100 = x 100 = 7.70%
( 100 + 96 ) /2 98
 Cost of preference share (redeemable under short cut method):
PD + ( f +d+ pr+ pi ) /n
K pd = x 100
( RV + NP ) /2
12 + (5+0+0+0 ) /10 12 + 0. 5
K da = x 100 = x 100 = 12. 82%
( 100 + 95 ) /2 97.50
 Cost of equity share (under growth rate approach):
D 2
Ke = +G= x 100+ 7% = 17 %
NP 20 ( 22−2 )

Determination of weighted average cost of capital (under book value weights)


Sources of capital Book value Book value K WACC
(in weights)(W) (after tax) (W x K)
Debentures 8,00,000 0.40 7.70 3.08
Preference shares 2,00,000 0.10 12.82 1.28
Equity shares 10,00,000 0.50 17.00 8.50
20,00,000 1.00 12.86%

Determination of weighted average cost of capital (under market value weights)


Sources of capital Market Market value K WACC
value (in weights)(W) (after tax) (W x K)
Debentures* 8,80,000 0.27 7.70 2.08
Preference shares** 2,40,000 0.07 12.82 0.90
Equity shares*** 22,00,000 0.66 17.00 11.22
33,20,000 1.00 14.20%

41 | P a g e
Working note: calculation of market values of capitals:
*Debenture capital = 8,000 x 110 = 8,80,000
**Preference share capital = 2,000 x 120 = 2,40,000
*** equity share capital = 1,00,000 x 22 = 22,00,000

23. The capital structure of Adamus Ltd in book value terms is as follows:
a. Equity capital (20 million shares, Rs. 10 par) Rs. 200 million
b. 12%, Preference capital (Rs. 100 each) Rs. 50 million
c. Retained earnings Rs. 350 million
d. Debentures 14% (Rs. 100 each) Rs. 120 million
e. Term loans, 13% Rs. 80 million
Total Rs. 800 million
The next expected dividend per share is Rs. 2.00. The dividend per share is
expected to grow at the rate of 12 per cent. The market price per share is Rs. 50.00.
Preference stock, redeemable after 10 years, is currently selling for Rs. 85 per share.
Debentures, redeemable after 5 years, are selling for Rs.90.00 per debenture. The tax
rate for the company is 30 per cent.
Calculate the average cost of capital.

Solution is as follows:
Calculation of specific cost of capitals:
1. Cost of equity (under growth rate method) and retained earnings
D1 2
Ke = x 100+ G = x 100 + 12 = 16 %
CMP 50
K e = K r = 16%
2. Cost of preference capital (redeemable) using short cut method
PD + ( f +d + pr+ pi ) /n
K pd = x 100
( RV + NP ) /2
12 + ( 0+15+0+0 ) /10 12 + 1. 5
K pd = x 100 = x 100 = 14 .60 %
( 100 + 85 ) /2 92.50
3. Cost of debt capital (redeemable) using short cut method
I ( 1−t ) + ( f +d+ pr+ pi ) /n
K da = x 100
( RV + NP ) /2
14 ( 1−0.30 ) + ( 0+10+0+0 ) /5 14 ( 0.70 ) + 2 11.8
K da = x 100 = x 100 = x 100 = 12.42 %
( 100 + 90 ) /2 95 95
4. Cost of term loan after tax = 13 (1-0.30) = 9.10%

42 | P a g e
Determination of weighted average cost of capital (under market value weights)
Sources of capital Market Market value K WACC
value (in weights)(W) (after tax) (W x K)
Equity capital & 1,000.0 m 0.81 16.00 12.96
retained earnings
Preference capital 42.5 m 0.03 14.60 0.44
Debentures 108.0 m 0.09 12.42 1.12
Term loan 80.0 m 0.07 9.10 0.64
1,230.5 m 1.00 WACC 15.15%

24. Susheel Corporation has the following book value capital structure:
Equity capital (Rs. 10 each) Rs. 100 million
11%, preference capital (Rs. 100 each) Rs. 10 million
Retained earnings Rs. 120 million
13.5%, Debentures (Rs. 100 each) Rs. 50 million
Term loans at 12% Rs. 80 million
Total Rs. 360 million
The next expected dividend per share is Rs. 1.50. The dividend per share is expected to
grow at the rate of 7 per cent. The market price per share is Rs. 20.00. Preference stock,
redeemable after 10 years, is currently selling for Rs. 75.00 per share. Debentures,
redeemable after 6 years, are selling for Rs. 80.00 per debenture. The tax rate for the
company is 50 per cent.
Calculate the WACC using book value and market value proportions.

Solution is as follows:
Calculation of specific cost of capitals:
1. Cost of equity (under growth rate method) and retained earnings
D1 1 .50
Ke = x 100+ G = x 100 + 7 = 14 .50 %
CMP 20
K e = K r = 14 .50 %
2. Cost of preference capital (redeemable) using short cut method
PD + ( f +d + pr+ pi ) /n
K pd = x 100
( RV + NP ) /2
11 + ( 0+25+0+0 ) /10 11 + 2.5
K pd = x 100 = x 100 = 15 .43 %
( 100 + 75 ) /2 87 . 50
3. Cost of debt capital (redeemable) using short cut method
I ( 1−t ) + ( f +d+ pr+ pi ) /n
K da = x 100
( RV + NP ) /2
13.5 ( 1−0 .50 ) + ( 0+20+0+0 ) /6 13.5 ( 0.50 ) + 3.33 10 .08
K da = x 100 = x 100 = x 100 = 11.20 %
( 100 + 80 ) /2 90 90
4. Cost of term loan after tax = 12 (1-0.50) = 6.00%

43 | P a g e
Determination of weighted average cost of capital (under market value weights)
Sources of capital Market Market value K WACC
value (in weights)(W) (after tax) (W x K)
Equity capital & 200.0 m 0.61 14.50 8.85
retained earnings
Preference capital 7.5 m 0.02 15.43 0.31
Debentures 40.0 m 0.12 11.20 1.34
Term loan 80.0 m 0.25 6.00 1.50
327.5 m 1.00 WACC 12.00%
25. Rao Corporation has a target capital structure of 60 per cent equity and 40 per cent
debt. Its cost of equity is 18 per cent and its pre-tax rate of debt is 13 per cent. if the
relevant tax rate is 35 per cent, what is WACC?

Solution is as follows:
WACC = [ W d x K d ( 1−t ) ] + [ W e x K e ]
WACC = [ 0 .4 x 0.13 ( 1−0. 35 ) ] + [ 0. 6 x 0. 18 ]
WACC = [ 0. 4 x 0.0845 ] + [ 0.108 ]
WACC = [ 0. 0338 ] + [ 0 .108 ] = 0. 1418 or 14. 18 %

26. From the following information of Excel Ltd., determine the WACC using (a) book value
weights and (b) market value weights. How are they different? Can you think of a
situation where the WACC would be the same using either of the weights?
Sources of finance Book value Market value Cost (%)
Equity capital 3,00,000 6,00,000 15
Retained earnings 1,00,000 13
Preference capital 50,000 60,000 8
Debt capital 2,00,000 1,90,000 6
Total 6,50,000 8,50,000

Determination of weighted average cost of capital (under market value weights)


Sources of capital Book value Book value K WACC
(in weights)(W) (after tax) (W x K)
Equity capital 3,00,000 0.46 15 6.90
Retained earnings 1,00,000 0.15 13 1.95
Preference capital 50,000 0.08 8 0.64
Debentures 2,00,000 0.31 6 1.86
6,50,000 1.00 WACC 11.35%

Determination of weighted average cost of capital (under market value weights)


Sources of capital Market Market value K WACC
value (in weights)(W) (after tax) (W x K)
*Equity capital 4,50,000 0.53 15 7.95

44 | P a g e
**Retained earnings 1,50,000 0.18 13 2.34
Preference capital 60,000 0.07 8 0.56
Debentures 1,90,000 0.22 6 1.32
8,50,000 1.00 WACC 12.17
Working note: equity share capital and retained earnings are in the ratio of 3:1 in book
value weights. Same ratio will apply to distribute Rs. 6,00,000 of market value among
equity share capital and retained earnings
*Equity capital = 6,00,000/4 * 3 = 4,50,000 and **RE = 6,00,000/4 * 1 = 1,50,000

27. A firm’s after tax cost of capital of the specific sources is as follows:
a. Cost of debt 8%
b. Cost of preference 14%
c. Cost of equity funds 17%
d. The following is the capital structure:
Debt capital Rs. 3,00,000
Preference capital Rs. 2,00,000
Equity capital Rs. 5,00,000
Total capital Rs. 10,00,000
e. Calculate the weighted average cost of capital using book value weights.

Solution is as follows:
Determination of weighted average cost of capital (under book value weights)
Sources of capital Book value Book value K WACC
(in weights)(W) (after tax) (W x K)
Equity capital 5,00,000 0.50 17 8.50
Preference capital 2,00,000 0.20 14 2.80
Debentures 3,00,000 0.30 8 2.40
10,00,000 1.00 WACC 13.70%

28. You are required to determine the weighted average cost of capital of ABC Ltd., using (i)
book value weights and (ii) market value weights. The following information is available
for your perusal:
Sources of capital Book value Market value Cost of capital
Equity 5,00,000 7,50,000 12%
Long-term debt 4,00,000 3,75,000 7%
Short-term debt 1,00,000 1,00,000 4%
Total 10,00,000 12,25,000
Solution is as follows:
Determination of weighted average cost of capital (under book value weights)
Sources of capital Book value Book value K WACC
(in weights)(W) (after tax) (W x K)
Equity capital 5,00,000 0.50 12 6.00
Long-term debt 4,00,000 0.40 7 2.80

45 | P a g e
Short-term debt 1,00,000 0.10 4 0.40
10,00,000 1.00 WACC 9.20%
Determination of weighted average cost of capital (under book value weights)
Sources of capital Book value Book value K WACC
(in weights)(W) (after tax) (W x K)
Equity capital 7,50,000 0.61 12 7.32
Long-term debt 3,75,000 0.31 7 2.17
Short-term debt 1,00,000 0.08 4 0.32
12,25,000 1.00 WACC 9.81%
29. A paper company has the following specific cost of capitals along with the indicated
book and market value weights
Source of capital Cost (%) Book value Market value
weights weights
Long-term debt 5 30 25
Preference shares 10 20 17
Equity shares 12 40 46
Retained earnings 12 10 12
a. Calculate the weighted average cost of capital using book value and market
value weights. Which of them do you consider better and why?

Solution is as follows:
Determination of weighted average cost of capital (under book value weights)
Sources of capital Book value K WACC
(in weights)(W) (after tax) (W x K)
Long-term debt 0.30 5 1.50
Preference shares 0.20 10 2.00
Equity shares 0.40 12 4.80
Retained earnings 0.10 12 1.20
1.00 WACC 9.50%

Determination of weighted average cost of capital (under book value weights)


Sources of capital Market value K WACC
(in weights)(W) (after tax) (W x K)
Long-term debt 0.25 5 1.25
Preference shares 0.17 10 1.70
Equity shares 0.46 12 5.52
Retained earnings 0.12 12 1.44
1.00 WACC 9.91%

46 | P a g e
30. XYZ Ltd., has the following book value capital structure:
Particulars Amount
Equity capital (Rs. 10 each fully paid) 15,00,00,000
12% preference shares (Rs. 100 each fully paid) 1,00,00,000
Retained earnings 20,00,00,000
11.5% debentures (Rs. 100 each) 10,00,00,000
11% term loans 12,50,00,000
The next expected dividend on equity shares per share is Rs. 3.60; the dividend per
share is expected to grow at the rate of 7%. The market price per share is Rs. 40.
Preference stock, redeemable after ten years, is currently selling at Rs. 75
per share. Debentures redeemable after 6 years are selling at Rs. 80 per debenture. The
income-tax rate for the company is 40 per cent.
you are required to calculate:
a. WACC using book value weights
b. Market value proportions
Solution is as follows:
Calculation of specific cost of capitals:
1. Cost of equity (under growth rate method) and retained earnings
D1 3.60
Ke = x 100+ G = x 100 + 7 = 16 .00 %
CMP 40
K e = K r = 16 .00 %
2. Cost of preference capital (redeemable) using short cut method
PD + ( f +d+ pr+ pi ) /n
K pd = x 100
( RV + NP ) /2
12 + ( 0+25+0+0 ) /10 12 + 2. 5
K pd = x 100 = x 100 = 16 .57 %
( 100 + 75 ) /2 87 .50
3. Cost of debt capital (redeemable) using short cut method
I ( 1−t ) + ( f +d + pr+ pi ) /n
K da = x 100
( RV + NP ) /2
11.5 ( 1−0. 40 ) + ( 0+20+0+0 ) /6 10 . 23
K da = x 100 = x 100 = 11. 37 %
( 100 + 80 ) /2 90
4. Cost of term loan after tax = 11 (1-0.40) = 6.60%
Determination of weighted average cost of capital (under market value weights)
47 | P a g e
Sources of capital Book value Book value K WACC
(in weights)(W) (after tax) (W x K)
Equity 15,00,00,000 0.26 16.00 4.16
Retained earnings 20,00,00,000 0.34 16.00 5.44
Preference capital 1,00,00,000 0.02 16.57 0.33
Debentures 10,00,00,000 0.17 11.37 1.93
Term loan 12,50,00,000 0.21 6.60 1.39
58,50,00,000 1.00 WACC 13.25%
Determination of weighted average cost of capital (under market value weights)
Sources of capital Book value Book value K WACC
(in weights)(W) (after tax) (W x K)
Equity 60,00,00,000 0.74 16.00 11.84
Preference capital 75,00,000 0.01 16.57 0.16
Debentures 8,00,00,000 0.10 11.37 1.14
Term loan 12,50,00,000 0.15 6.60 0.99
81,25,00,000 1.00 WACC 14.13%

31. Determine the weighted marginal cost of capital schedule for the company, if it raises
Rs. 10 crore next year, given the following information:
a. The amount will be raised by equity and debt in equal proportions
b. The company expected to retain Rs. 1.5 crore earnings next year.
c. The additional issue of equity shares will result in the net price per share being
fixed at Rs. 32. Dividend per share is expected to grow at the rate of 7%.
Opportunity cost for the company is 16%. Dividend per share is Rs. 3.60.
d. The debt capital raised by way of term loans will cost 12 per cent for the first Rs.
2.5 crore and 13 per cent for the next Rs. 2.5 crore. Tax rate is at 40%

Solution is as follows:
Determination of weighted average cost of capital (under market value weights)
Sources of capital Market Market value K WACC
value (inweights)(W) (after tax) (W x K)
Equity:
a. Share capital 3,50,00,000 0.35 18.25* 6.39
b. Retained earnings 1,50,00,000 0.15 16.00 2.40
Debt:
a. 12% loan 2,50,00,000 0.25 7.20** 1.80
b. 13% loan 2,50,00,000 0.25 7.80*** 1.95
10,00,00,000 1.00 WACC 12.54%

Working note:

48 | P a g e
D 3.60
¿ Ke = x 100 + g = x 100 + 7 = 18 .25%
CMP 32
** K da = I [ 1−t ] = 12 [ 1−0. 40 ] = 7 .20%
*** K da = I [ 1−t ] = 13 [ 1−0.40 ] = 7.80 %

32. The following information is available for you perusal:


Company’s present book values structure is as follows:
Debentures (Rs. 100 per debenture) Rs. 7,00,000
Preference shares (Rs. 100 per share) Rs. 3,00,000
Equity shares (Rs. 10 each) Rs. 10,00,000
All these securities are traded in the capital market and their recent prices are:
Debentures Rs. 110 each; preference share Rs. 120 each; and equity share Rs. 22
each. Anticipated external financing opportunities are:
a. Rs. 100, redeemable debentures at face value after 8 years, 13 per cent interest rate,
4% floatation cost.
b. 14% redeemable preference shares (5 years), it involves a floatation cost of 5 per
cent and the sales price Rs. 100
c. Equity share: Rs. 2 per share brokerage; Rs. 22 per share selling price
In addition, the dividend expected as the equity share at the end of the year is Rs. 2 per
share. The anticipated growth rate in dividends is 6 per cent and the firm has the
practice of paying all its earnings in the form of dividends. The corporate tax rate is 35%.
You are required to determine the WACC using book value and market
value methods
Solution is as follows:
Initially we need to determine the specific cost of all the sources of capitals:
 Cost of debt capital (redeemable under short cut method):
I ( 1−t ) + ( f +d+ pr+ pi ) /n
K da = x 100
( RV + NP ) /2
13 ( 1−0.35 ) + ( 4+0+0+0 ) /8 8.45 + 0 .5
K da = x 100 = x 100 = 9.13 %
( 100 + 96 ) /2 98
 Cost of preference share (redeemable under short cut method):
PD + ( f +d+ pr+ pi ) /n
K pd = x 100
( RV + NP ) /2
14 + ( 5+0+0+0 ) /10 14 + 0.5
K da = x 100 = x 100 = 14. 87 %
( 100 + 95 ) /2 97 .50
 Cost of equity share (under growth rate approach):

49 | P a g e
D 2
Ke = +G= x 100+ 6% = 16%
NP 20 ( 22−2 )

Determination of weighted average cost of capital (under book value weights)


Sources of capital Book value Book value K WACC
(in weights)(W) (after tax) (W x K)
Debentures 7,00,000 0.35 9.13 3.20
Preference shares 3,00,000 0.15 14.87 2.23
Equity shares 10,00,000 0.50 16.00 8.00
20,00,000 1.00 13.43%
Determination of weighted average cost of capital (under market value weights)
Sources of capital Market Market value K WACC
value (in weights)(W) (after tax) (W x K)
Debentures* 7,70,000 0.23 9.13 2.10
Preference shares** 3,60,000 0.11 14.87 1.64
Equity shares*** 22,00,000 0.66 16.00 10.56
33,30,000 1.00 14.30%

Working note: calculation of market values of capitals:


*Debenture capital = 7,000 x 110 = 7,70,000
**Preference share capital = 3,000 x 120 = 3,60,000
*** equity share capital = 1,00,000 x 22 = 22,00,000

33. The shares of a company are selling at Rs. 20 per share. The firm had paid dividend at
Rs. 2 per share last year. The estimated growth of the company is approximately 5 per
cent per year:
a. Determine the cost of equity capital of the company.
b. Determine the estimated market price of the equity shares if the anticipated
growth rate of the firm rises to 8% and falls to 3%.

Solution is as follows:
1. Cost of equity capital:
D 2 .10
K e = 1 x 100 + G = x 100 + 5 = 15 .5%
CMP 20
2. Market price of the equity if growth rises to 8%
D 2.16 2.16
Po = = = = Rs . 28.80
[ K e − g ] [ 0. 155 − 0. 08 ] 0. 075
3. Market price of the equity if growth falls to 3%
D 1.94 1.94
Po = = = = Rs . 10 .49
[ K e + g ] [ 0 .155 + 0 .03 ] 0 .185

50 | P a g e
34. The Evergreen investment company manages a stock fund consisting of four stocks with
the following market values and beta:
Stoc Market values Beta
k
A 2,00,000 1.16
B 1,00,000 1.20
C 1,50,000 0.80
D 50,000 0.50
If the risk free rate of interest is 9% and the market return is 15%, what is the portfolio’s
expected return?

Solution is as follows:
Initially we need to determine the portfolio beta of all the four stocks taking
investment as the weightage as we need to determine portfolio return of investment.
Weightage of investment:
2,00,000
Stock A = = 0.40
5,00,000
1,00,000
Stock B = = 0.20
5,00,000
1,50 ,000
Stock C = = 0.30
5,00 ,000
50,000
Stock D = = 0.10
5,00,000

To calculate the fund’s expected return, the funds beta should be estimated
N
β p = ∑ X 1 βi
i=1
Where X 1 is the weight of each security and βi is the beta co−eficient for the security
β p = [ . 4 x 1. 16 ] + [ . 2 x 1 .2 ] + [ . 3 x . 8 ] + [ . 1 x . 5 ]
β p = 0 . 464 + 0 .24 + 0 . 24 + 0 .05 = 0 . 994

51 | P a g e
Portfolio return as CAPM
Rp = Rf + β p [ Rm − Rf ]
R p = 9 + 0.994 [ 15 − 9 ]
R p = 14 .96

35. The Broadway investment company managers equity fund consisting of five stocks with
the following market values and beta’s
Stoc Market values Beta
k
A 3,00,000 1.3
B 2,50,000 1.2
C 2,00,000 0.9
D 1,50,000 0.5
E 1,00,000 1.6
If the risk free rate of interest is 9% and the market return is 16%, what is the portfolio’s
expected return?

Solution is as follows:
Initially we need to determine the portfolio beta of all the four stocks taking
investment as the weightage as we need to determine portfolio return of investment.
Weightage of investment:
3,00,000
Stock A = = 0.30
10,00,000
2,50,000
Stock B= = 0.25
10 ,00,000
2,00,000
Stock C= = 0.20
10 ,00,000
1,50,000
Stock D= = 0.15
10,00,000
1,00,000
Stock D= = 0.10
10,00,000

To calculate the fund’s expected return, the funds beta should be estimated

52 | P a g e
N
β p = ∑ X 1 βi
i=1
Where X 1 is the weight of each security and βi is the beta co−eficient for the security
β p = [ 0 .30 x 1. 30 ] + [ 0 . 25 x 1 . 20 ] + [ 0 .20 x 0 . 90 ] + [ 0 .15 x 0 .50 ] + [ 0 . 10 x 1 . 60 ]
β p = [ 0. 39 + 0. 30 + 0 .18 + 0 .075 + 0 .16 ] = 1 .105

Portfolio return as CAPM


Rp = Rf + β p [ Rm − Rf ]
R p = 9 + 1.105 [ 16 − 9 ]
R p = 16.735 %

36. Mr. Azad holds the following:


Stoc Market values Beta
k
A 3,00,000 0.60
B 1,80,000 1.00
C 1,20,000 1.20
What is the expected rate of return on his portfolio if the risk free rate is 6% and the
expected return on market portfolio is 15%

Solution is as follows:
Initially we need to determine the portfolio beta of all the four stocks taking
investment as the weightage as we need to determine portfolio return of investment.
Weightage of investment:
3,00,000
Stock A = = 0.50
6 ,00,000
1,80,000
Stock B = = 0.30
6,00,000
1,20,000
Stock C = = 0.20
6 ,00,000

To calculate the fund’s expected return, the funds beta should be estimated
N
β p = ∑ X 1 βi
i=1
Where X 1 is the weight of each security and βi is the beta co−eficient for the security
β p = [ 0 .50 x 0 . 60 ] + [ 0 .30 x 1. 00 ] + [ 0 . 20 x 1 .20 ]
β p = [ 0. 30 + 0 .30 + 0 .24 ] = 0. 84

53 | P a g e
Portfolio return as CAPM
Rp = Rf + β p [ Rm − Rf ]
R p = 6 + 0 .84 [ 15 − 6 ]
R p = 13 .56 %

Solutions on Capital Budgeting Decisions


1. A company is considering two mutually exclusive projects. Both require an initial cash outlay of
Rs. 10,000 each, and have a life of five years. The company’s required rate of return is 10 per
cent and is in tax bracket of 50 per cent. The projects will be depreciated on a straight line basis.
The before tax cash flows expected o be generated by the projects are as follows:
Project Before tax cash flows
s 1 2 3 4 5
A 4,000 4,000 4,000 4,000 4,000
B 6,000 3,000 2,000 5,000 5,000
Calculate the each project: payback period, average rate of return, net present value,
profitability index and internal rate of return.

Solution is as follows:
Calculation of Pay Back Period:
Inflows = Profit after tax + Depreciation
The given inflows are before tax cash flows. So, inflows will be calculated as follows:
PAT = PBT – TAX = PAT + Depreciation = Inflows
cos t of the asset 10,000
Depreciation = = = Rs.2,000 per year
Life of the asset 5
Year PBT Tax PAT Dept inflows PV@10% PV of cash
s inflows
1 4,000 2,000 2,000 2,000 4,000 0.909 3,636
2 4,000 2,000 2,000 2,000 4,000 0.826 3,304
3 4,000 2,000 2,000 2,000 4,000 0.751 3,004
4 4,000 2,000 2,000 2,000 4,000 0.683 2,732
5 4,000 2,000 2,000 2,000 4,000 0.621 2,484
10,00 15,160
0
When cashflows are even for all the year, then PBP will be as follows:

54 | P a g e
Initial investment 10,000
PBP = = = 2.5 years
Average cash inf lows 4 ,000

Average inflows 0 ∗¿ Total PAT 10, 0 Initalinvestment 10, 0


AR = x 10 =2, x10 =40%¿Average inflows= = =Rs.2,0 ∗¿ Average investment = = =Rs.5,0 *
Averageinvestment 5,0 * No.of years 5 2 2
NPV = Total PV of cash inf lows − Initial investment
NPV = 15,160 = 10,000 = 5,160
PV of cash inf lows 15,160
Pr ofitability index = = = 1.516
Initial investment 10,000
IRR will be calculated in four steps:
a. Find PBP
b. Refer Table 4 (PVIFA) and find where PBP falls.
c. Consider two discount rates: LDF and HDF
d. Convert the inflows into PV@LDF and PV@HDF
e. Calculation of IRR
Step 1
Initialinvestment 10,000
PBP = = = 2.5 years
average cashflows 4 ,000
Step 2
By referring Table 4 (PVIFA) PBP of 2.5 years falls in between 28% and 32% in 5 year. Now let
us find PV at both the discount factors as follows:
Step 3
As inflows (PATBD) are equal for all the years, we can find the product by taking PVIFA
PVIFA at 28% = 4,000 x 2,532 = 10,128
PVIFA at 32% = 4,000 x 2.345 = 9,380
Step 4
As investment of Rs. 10,000 falls in between 28% and 32%, by applying IRR we can
determine the exact discount factor:
[ PV @ LDF − II ]
IRR = LDF +
[
[ PV @ LDF − PV @ HDF ]
x [ HDF − LDF ]
]
[ 10, 128 − 10 ,000 ]
IRR = 28 +
[
[ 10, 128 − 9 , 380 ] ]
x [ 32 − 28 ]

128
IRR = 28 +
748[ ]x4
IRR = 28 + 0 .68 = 28 .68 %

55 | P a g e
2. A company if considering expanding its production. It can go in either for an automatic machine
costing Rs. 2,24,000 with an estimated life of 5 years or an ordinary machine costing Rs. 60,000
having an estimated life of 8 years. The annual sales and costs are estimated as follows:
Particulars Automatic Ordinary
machine Machine
Sales 1,50,000 1,50,000
Costs:
Materials 50,000 50,000
Labour 12,000 60,000
Variable 24,000 20,000
overheads
Calculate payback period and average rate of return

Solution is as follows:
Firstly we need to determine the inflows for both the machines in the following manner:
Particulars Automatic Ordinary
machine machine
Sales 1,50,000 1,50,000
Less:
Materials 50,000 50,000
Labour 12,000 60,000
Variable overheads 24,000 20,000
PBDT 64,000 20,000
Less: Depreciation 44,800 7,500
PBT 19,200 12,500
Less: Tax 9,600 6,250
PAT 9,600 6,250
Add: Depreciation 44,800 7,500
PATBD (inflows) 54,400 13,750

56 | P a g e
Pay Back Period for Automatic Machine:
Initial investment 2,24,000
PBP = = = 4.12 years
Average annual cashflows 54,400
Pay Back Period for Ordinary Machine:
Initial investment 60,000
PBP = = = 4.36 years
Average annual cashflows 13,750
ARR for Automatic Machine:
Average income 9,600
ARR = x 100 = x 100 = 8.57%
Average ivestment 1,12,000
ARR for Ordinary Machine:
Average income 6,250
ARR = x 100 = x 100 = 20.83%
Average ivestment 30,000

3. A project costs Rs. 20,00,000 and yields annually profit of Rs. 3,00,000 after depreciation at
12.50 per cent per year but before tax at 50 per cent. Calculate payback period and suggest
whether it should be accepted or rejected based on 6 year standard payback period.

Solution is as follows:
Firstly we need to determine Inflows of the machine:
Inflows = PATBD
Given inflows are PBT
Particulars Amount
PBT 3,00,000
Less: tax 1,50,000
PAT 1,50,000
Add: Depreciation 2,50,000
PATBD 4,00,000
Initial Investment 20,00 ,000
PBP = = = 5 years
Average cash inf lows 4 ,00,000
Project has to be accepted as its pay back period is less than the standard of 6 years.

4. XYZ Ltd. is considering two projects. Each project required an investment of Rs. 10,000. The
firm’s cost of capital is 10 per cent. The net cash inflows in two projects X and Y are as follows:
Year 1 2 3 4 5
s
X 5,000 4,000 3,000 1,000 --------
Y 1,000 2,000 3,000 4,000 5,000
The company has fixed 3 years PBP as the cut-off point. State which project should be accepted?

Solution is as follows:
Inflows of the project are of uneven cash inflows. In that case we need to determine the
cumulative cash inflows of the project.

57 | P a g e
Project A Project B
Years Cash flows Cumulative Years Cash flows Cumulative
cash flows cash flows
1 5,000 5,000 1 1,000 1,000
2 4,000 9,000 2 2,000 3,000
3 3,000 12,000 3 3,000 6,000
4 1,000 13,000 4 4,000 10,000
5 5,000 15,000
required cash flows required cash flows
PBP = Minimum time + PBP = Minimum time +
Annaul cash flows Annaul cash flows
1 , 000 PBP = 4 years
PBP = 2 + = 2 + 0 .33 = 2 .33 years
3 , 000
Project A has to be chosen as its pay back is less than project B

5. A limited company has under consideration the following two projects. Their details are as
follows:
Particulars Project X Project Y
Investment in machinery 10,00,000 15,00,000
Working capital 5,00,000 5,00,000
Life of machinery 4 years 6 years
Scrap value of machinery 10 per cent 10per cent
Tax rate 50 per cent 50 per cent
Income before depreciation and tax at the end of the every year is as follows:
Yea 1 2 3 4 5 6
r
X 8,00,000 8,00,000 8,00,000 8,00,000 ------ --------
Y 15,00,000 9,00,000 15,00,000 8,00,000 6,00,000 3,00,000
You are required to calculate the average rate of return and suggest which project is to be
preferred.
Solution is as follows:
Inflows = PAT
Given inflows are profits before dept and tax
When working capital and scrap value are given ARR formula is as follows:
Project X
Year PBDT Less: PBT Less: Tax PAT
s Dept
1 8,00,000 2,25,000 5,75,000 2,87,500 2,87,500
2 8,00,000 2,25,000 5,75,000 2,87,500 2,87,500
3 8,00,000 2,25,000 5,75,000 2,87,500 2,87,500
4 8,00,000 2,25,000 5,75,000 2,87,500 2,87,500
Total profits 11,50,000

Total cos t − scrap value 10,00,000 − 1,00,000


Depreciation = = = 2,25,000
life of the asset 4

58 | P a g e
Average income
ARR = x 100
Average investment
500∗¿ Total income 11,50,000
ARR = 2,87, x 100 = 27.38% ¿¿ Average income = = = 2,87,500
10,50,000** No of years 4
Initial investment − Scrap value
** Average investment = + Working capital + scrap value
2
10,00,000 − 1,00,000
Average investment = + 5,00,000 + 1,00,000
2
9,00,000
Average investment = + 6,00,000 = 10,50,000
2

Project X
Year PBDT Less: PBTLess: Tax PAT
s Dept
1 15,00,000 2,25,000 12,75,000 6,37,500 6,37,500
2 9,00,000 2,25,000 6,75,000 3,37,500 3,37,500
3 15,00,000 2,25,000 12,75,000 6,37,500 6,37,500
4 8,00,000 2,25,000 5,75,000 2,87,500 2,87,500
5 6,00,000 2,25,000 3,75,000 1,87,500 1,87,500
6 3,00,000 2,25,000 75,000 37,500 37,500
Total profits 21,25,000

Total cos t − scrap value 15,00,000 − 1,50,000


Depreciation = = = 2,25,000
life of the asset 6
Average income
ARR = x 100
Average investment
167∗¿ Total income 21,25,000
ARR = 3,54, x 100 = 26.73% ¿¿ Average income = = = 3,54,167
13,25,000** No of years 6
Initial investment − Scrap value
** Average investment = + Working capital + scrap value
2
15,00,000 − 1,50,000
Average investment = + 5,00,000 + 1,50,000
2
13,50,000
Average investment = + 6,50,000 = 13,25,000
2

59 | P a g e
6. A project costs Rs. 5,00,000 and has a scrap value of Rs. 1,00,000. Its stream of income before
depreciation and taxes during first year through five years is Rs. 1,00,000, Rs. 1,20,000, Rs.
1,40,000, Rs. 1,60,000 and Rs. 2,00,000. Assume a 50 per cent tax rate and depreciation on
straight line basis. Calculate the accounting rate for the project.

Solution is as follows:
We need to determine inflows (PAT)
Project X
Year PBDT Less: PBT Less: Tax PAT
s Dept
1 1,00,000 80,000 20,000 10,000 10,000
2 1,20,000 80,000 40,000 20,000 20,000
3 1,40,000 80,000 60,000 30,000 30,000
4 1,60,000 80,000 80,000 40,000 40,000
5 2,00,000 80,000 1,20,000 60,000 60,000
Total profits 1,60,000

Total cos t − scrap value 15,00,000 − 1,50,000


Depreciation = = = 2,25,000
life of the asset 6

60 | P a g e
Average income
ARR = x 100
Average investment
000∗¿ Total income 1,60,000
ARR =32, x 100=12.80%¿¿Average income= = =32,000
2,50,000** No of years 5
Initial investment
**Average investment=
2
5,00,000
Average investment =
2
5,00,000
Average investment = =2,50,000
2

7. From the following data determine the accounting rate of return:


Particulars Machine A Machine B
Original cost 56,125 56,125
Additional investment in working capital 5,000 6,000
Estimated life (years) 5 5
Estimated salvage value 3,000 3,000
Income tax rate 55 55

Cash flows (PAT) are as follows:


1 2 3 4 5
Machine
Machine A 3,375 5,375 7,37 9,375 11,375
5
Machine B 11,375 9,375 7,37 5,735 3,375
5

Solution is as follows:
Machine A Machine B

61 | P a g e
Averageincome Averageincome
ARR= x100 ARR= x100
Averageinvestment Averageinvestment
375∗¿ Total income 36,875 375∗¿ Total income 36,875
ARR=7, x100=21.34%¿¿AI= = =7,375 ARR=7, x 100=20.74%¿¿AI= = =7,375
34,562.50** Noof years 5 35,562.50** No of years 5
I I−SV I I−SV
**A I = +WC+SV **A I = +WC+SV
2 2
56,125−3,000 56,125−3,000
AI= +5,000+3,000 AI= +6,000+3,000
2 2
53,125 53,125
AI= +8,000=34,562.50 AI= +9,000=35,562.50
2 2
8. Calculate the NPV from the following cash streams:
Years 0 1 2 3 4 5
Cash flow 10,00,000 ----- ------ ------ ------- ------
Cash flow --- 2,00,00 200,000 3,00,00 3,00,000 3,50,000
0 0
The cost of capital of the firm is 10 per cent.

Solution is as follows:
NPV = PV of cash inflows – Initial Investment
Year Cash flows PV@10% PV of inflows
s
1 2,00,000 0.909 1,81,800
2 2,00,000 0.826 1,65,200
3 3,00,000 0.751 2,25,300
4 3,00,000 0.683 2,04,900
5 3,50,000 0.621 2,17,350
Total PV of cash inflows 9,94,550
Less: initial investment 10,00,000
Negative NPV (5,450)
9. No project is acceptable unless the yield is 10 per cent. Cash inflows of a certain project along
with cash outflows are given below:
Years 0 1 2 3 4 5
Cash flow 1,50,00 30,000 ------ ------ ------- ------
Cash flow 0 20,000 30,00 60,000 80,00 30,000
---- 0 0
The salvage value at the end of 5th year is Rs. 40,000. Calculate the NPV and PI

Solution is as follows:
Years Inflows PV@10% PV of cash flows
1 20,000 0.909 18,180
2 30,000 0.826 24,780
3 60,000 0.751 45,060
4 80,000 0.683 54,640

62 | P a g e
5 30,000 0.621 18,630
Total PV of cash flows 1,61,290
Less: II [1,50,000 + 30,000 (0.909)] 1,77,270
Negative NPV (15,980)

10. A project needs an investment of Rs. 1,38,500. The cost of capital is 12 per cent. The net cash
inflows are as follows:
Year 1 2 3 4 5
Cash flows 30,000 40,00 60,000 30,000 20,000
0
Calculate IRR and suggest whether project should be accepted or not.

Solution is as follows:
Calculation of IRR involves four steps:
Step 1
Initialinvestment 1,38,500
PBP = = = 3.847 years
average cashflows 36,000
Step 2
By referring Table 4 (PVIFA) PBP of 3.847 years falls in between 8% and 10% in 5 th year row.
Now let us find PV at both the discount factors as follows:
Step 3
Year PV @ PV of cash PV @ PV of cash
Cash flows
s 8% Flows @ LDF 10% Flows @ HDF
1 30,000 0.926 27,780 0.909 27,270
2 40,000 0.857 34,280 0.826 33,040
3 60,000 0.794 47,640 0.751 45,060
4 30,000 0.735 22,050 0.683 20,490
5 20,000 0.681 13,620 0.621 12,420
1,45,370 1,38,280

Step 4
As investment of Rs. 10,000 falls in between 28% and 32%, by applying IRR we can
determine the exact discount factor:

63 | P a g e
[ PV @ LDF − II ]
IRR = LDF +
[
[ PV @ LDF − PV @ HDF ] ]x [ HDF − LDF ]

[ 1 , 45 , 370 − 1 ,38 , 500 ]


IRR = 8 +
[ ]
[ 1 , 45 , 370 − 1 ,38 , 280 ]
x [ 10 − 8 ]

6 ,870
IRR = 8 + [ ]
7 ,090
x2
IRR = 8 + 1 . 94 = 9 . 94 %

11. Calculate IRR for the project having the following cash streams
Years 0 1 2 3 4
Cash flow 1,00,000 ----- ----- ---- -------
Cash flow ---- 30,000 30,00 40,000 45,000
0

Solution is as follows:
Calculation of IRR involves four steps:
Step 1
Initialinvestment 1,00,000
PBP = = = 2.759 years
average cashflows 36,250
Step 2
By referring Table 4 (PVIFA) PBP of 2.759 years falls in between 16% and 20% in 4 th year row.
Now let us find PV at both the discount factors as follows:
Step 3
Year PV of cash PV of cash
Cash flows PV @ 16% PV @ 20%
s Flows @ LDF Flows @ HDF
1 30,000 0.862 25,860 0.833 24,990
2 40,000 0.743 29,720 0.694 27,760
3 40,000 0.641 25,640 0.579 23,160
4 45,000 0.552 24,840 0.482 21,690
1,06,060 97,600
Step 4
As investment of Rs. 1,00,000 falls in between 16% and 20%, by applying IRR we can
determine the exact discount factor:
PV @ LDF − II ]
IRR = LDF +
[[[
PV @ LDF − PV @ HDF ] ] x [ HDF − LDF ]

[ 1, 06 . 060 − 1, 00 , 000 ]
IRR = 16 +
[
[ 1, 06 , 060 − 97, 600 ] ] x [ 20 − 16 ]

6, 060
IRR = 16 + [ ]
8, 460
x4
IRR = 16 + 2 . 87 = 18 , 87%

12. Consider two projects X and Y, being evaluated by a firm that has a cost of capital of 10 per cent.
calculate IRR and NPV for the projects:

64 | P a g e
Year C0 C1 C2 C3 C4
Project X 1,10,000 31,000 40,000 50,00 70,000
Project Y 1,10,000 71,000 40,000 0 20,000
40,00
0
Solution is as follows:
PROJECT X
Calculation of NPV:
NPV = PV of cash inflows – initial investment
NPV = *1,59,029 – 1,10,000 = 49,029
Year C1 C2 C3 C4
Project X 31,000 40,00 50,000 70,000
0
PV @ 10% 0.909 0.826 0.751 0.683
*PV of cash flows 28,179 33,04 50,000 47,810 *1,59,029
0

Calculation of IRR involves four steps:


Step 1
Initial investment 1,10,000
PBP = = = 2.304 years
average cashflows 47,750
Step 2
By referring Table 4 (PVIFA) PBP of 2.304 years falls in between 22% and 24% in 4 th year row.
Now let us find PV at both the discount factors as follows:
Step 3
Year Cash flows PV @ 22% PV of cash PV @ 24% PV of cash
s Flows @ LDF Flows @ HDF
1 31,000 0.820 25,420 0.806 24,986
2 40,000 0.672 26,880 0.650 26,000
3 50,000 0.551 27,550 0.524 26,200
4 70,000 0.452 31,640 0.423 29,610
1,11,490 1,06,796
Step 4
As investment of Rs. 1,10,000 falls in between 22% and 24%, by applying IRR we can
determine the exact discount factor:
[ PV @ LDF − II ]
IRR = LDF +
[
[ PV @ LDF − PV @ HDF ] ] x [ HDF − LDF ]

[ 1 , 11, 490 − 1 ,10 , 000 ]


IRR = 22 +
[ ]
[ 1 , 11, 490 − 1 ,06 , 796 ]
x [ 24 − 22 ]

1, 490
IRR = 22 + [ ]
4, 694
x2
IRR = 22 + 0 . 63 = 22. 63 %

65 | P a g e
PROJECT Y
Calculation of NPV:
NPV = PV of cash inflows – initial investment
NPV = *1,41,279 – 1,10,000 = 31,279
Year C1 C2 C3 C4
Project X 71,000 40,00 40,000 20,000
0
PV @ 10% 0.909 0.826 0.751 0.683
*PV of cash flows 64,539 33,04 30,040 13,660 *1,41,279
0

Calculation of IRR involves four steps:


Step 1
Initial investment 1,10,000
PBP = = = 2.573 years
average cashflows 42,750
Step 2
By referring Table 4 (PVIFA) PBP of 2.573 years falls in between 20% and 26% in 4 th year row.
Now let us find PV at both the discount factors as follows:
Step 3
Year Cash flows PV @ 22% PV of cash PV @ 26% PV of cash
s Flows @ LDF Flows @ HDF
1 71,000 0.820 58,220 0.794 56,374
2 40,000 0.672 26,880 0.630 25,200
3 40,000 0.551 22,040 0.500 20,000
4 20,000 0.452 9,040 0.397 7,940
1,16,180 1,09,514
Step 4
As investment of Rs. 1,10,000 falls in between 22% and 26%, by applying IRR we can
determine the exact discount factor:
[ PV @ LDF − II ]
IRR = LDF +
[
[ PV @ LDF − PV @ HDF ] ] x [ HDF − LDF ]

[ 1, 16 ,180 − 1 , 10 ,000 ]
IRR = 22 +
[ ]
[ 1, 16 ,180 − 1 , 09 ,514 ]
x [ 26 − 22 ]

6 ,180
IRR = 22 + [ ]
6 ,666
x4
IRR = 22 + 3.708 = 25 . 708 %

66 | P a g e
13. A company has to select one of the following two projects:
Year 0 1 2 3 4
Project X 11,000 6,000 2,000 1,000 5,000
Project Y 10,000 1,000 1,000 2,000 10,000
Calculate IRR. Suggest the best alternative on the above basis.

Solution is as follows:
Project Y
Calculation of IRR involves four steps:
Step 1
Initial investment 11,000
PBP = = = 3.143 years
average cashflows 3,500
Step 2
By referring Table 4 (PVIFA) PBP of 3.143 years falls in between 10% and 12% in 4 th year row.
Now let us find PV at both the discount factors as follows:
Step 3
Year Cash PV @ 10% PV of cash PV @ 12% PV of cash
s flows Flows @ LDF Flows @ HDF
1 6,000 0.909 5,454 0.893 5,358
2 2,000 0.826 1,652 0.797 1,594
3 1,000 0.751 751 0.712 712
4 5,000 0.683 3,415 0.636 3,180
11,272 10,844
Step 4
As investment of Rs. 11,000 falls in between 10% and 12%, by applying IRR we can
determine the exact discount factor:
[ PV @ LDF − II ]
IRR = LDF +
[
[ PV @ LDF − PV @ HDF ] ] x [ HDF − LDF ]

[ 11, 272 − 11, 000 ]


IRR = 10 +
[ ]
[ 11, 272 − 10 ,844 ]
x [ 12 − 10 ]

272
IRR = 10 + [ ]
428
x2
IRR = 10 + 1 .27 = 11.27 %

PROJECT Y
Calculation of IRR involves four steps:
Step 1
Initial investment 10,000
PBP = = = 2.857 years
average cashflows 3 ,500
Step 2

67 | P a g e
By referring Table 4 (PVIFA) PBP of 2.857 years falls in between 10% and 12% in 4 th year row.
Now let us find PV at both the discount factors as follows:

Step 3
Year Cash PV of cash PV of cash
PV @ 10% PV @ 12%
s flows Flows @ LDF Flows @ HDF
1 1,000 0.909 909 0.893 893
2 1,000 0.826 826 0.797 797
3 2,000 0.751 1,502 0.712 1,424
4 10,000 0.683 6,830 0.636 6,360
10,067 9,474
Step 4
As investment of Rs. 10,000 falls in between 10% and 12%, by applying IRR we can
determine the exact discount factor:
[ PV @ LDF − II ]
IRR = LDF +
[
[ PV @ LDF − PV @ HDF ] ] x [ HDF − LDF ]

[ 10 , 067 − 10 , 000 ]
IRR = 10 +
[
[ 10 , 067 − 9 , 474 ]] x [ 12 − 10 ]

67
IRR = 10 + [ ]
593
x4
IRR = 10 + 0 . 452 = 10. 452 %

68 | P a g e
14. X ltd. is considering the purchase of a new machine, which will carry out some operations at
present performed by labour. Two alternative models, A and B are available for the purpose.
From the following information, prepare a profitability statement for submission to the
management and calculate PBP.

Particulars Machine A Machine B


Estimated life (years) 5 6
Cost of machine 80,000 1,50,000
Estimated additional cost:
Indirect material (p.a) 2,000 3,000
Maintenance (p.m) 500 750
Supervision (per quarter) 3,000 4,500
Estimated savings:
a. Workers not required 10 15
b. Wages per worker (pa) 7,200 7,200
Savings in scrap 8,000 12,000
Depreciation is calculated under straight line method. Taxation may be taken at 50 per cent of
net profit.

Solution is as follows:
In the given question we need to determine cash inflows by taking savings and expenditure
into account.
Particulars Machine A Machine B
Estimated savings:
 Savings in wages (10x7200) 72,000 1,08,000
 Savings in scrap (15x7200) 8,000 12,000
Total savings 80,000 1,20,000
Estimated additional cost:
 Indirect material 2,000 3,000
 Maintenance 6,000 9,000
 Supervision 12,000 18,000
Total additional cost 20,000 30,000
Net savings 60,000 90,000
Less: Depreciation 14,400 23,000
PBT 45,600 67,000
Less: tax 22,800 33,500
PAT 22,800 33,500
Add: depreciation 14,400 23,000
PATBD (inflow) 37,200 56,500
Calculation of depreciation:
cost − SV 14,400 23,000
Dept=
Life
Payback period

69 | P a g e
Initial investment 80,000
=2.15 yrs
1,50,000
= 2.65 yrs
PBP = 37,200 56,500
Average cash inf lows
15. Ramnath washing machines ltd is considering the purchase of a new machine which would carry
out some operations performed by manual labour. A and B are alternative models. The
following information is available. Using payback period, identify the most profitable
investment.
Particulars Machine A Machine B
Cost of machine 15,000 25,000
Estimated life of the machine 5 years 6 years
Estimated cost of indirect labour (pa) 600 800
Estimated savings in scrap 1,000 1,500
Estimated savings in direct wages:
a. Workers not required 15 20
b. Wages per worker 600 600
Additional cost of the maintenance (pa) 700 1,100
Additional cost of supervision (pa) 1,200 1,600
Assume a tax rate of 50 per cent

Solution is as follows:
In the given question we need to determine cash inflows by taking savings and expenditure
into account.
Particulars Machine A Machine B
Estimated savings:
 Savings in wages 9,000 12,000
 Savings in scrap 1,000 1,500
Total savings 10,000 13,500
Estimated additional cost:
 Indirect labout 600 800
 Maintenance 700 1,100
 Supervision 1,200 1,600
Total additional cost 2,500 3,500
Net savings 7,500 10,000
Less: Depreciation 2,800 3,917
PBT 4,700 6,083
Less: tax 2,350 3,041
PAT 2,350 3,042
Add: depreciation 2,800 3,917
PATBD (inflow) 5,150 6,959
Calculation of depreciation: 15,000 − 1,000 25,000 − 1,500
cost − SV 5 6
Dept=
Life = 2,800 = 3,917
Payback period
Initial investment 15,000
=2.91 yrs
25,000
= 3.59 yrs
PBP = 5 ,150 6 ,959
Average cash inf lows

70 | P a g e
16. KMF wants to replace the manual operations by a new machine. There are two alternative
models, A and B of the new machine. Using payback period, suggest the most profitable
investment:
Particulars Machine A Machine B
Cost of the machine 9,000 18,000
Estimated life of the machine 4 years 5 yeas
Estimated savings in scrap 500 800
Estimated savings in direct wages 6,000 8,000
Additional cost of the maintenance 800 1,000
Additional cost of supervision 1,200 1,800
Ignore taxation

Solution is as follows:
In the given question we need to determine cash inflows by taking savings and expenditure
into account.
Particulars Machine A Machine B
Estimated savings:
 Savings in wages 6,000 8,000
 Savings in scrap 500 800
Total savings 6,500 8,800
Estimated additional cost:
 Maintenance 800 1,000
 Supervision 1,200 1,800
Total additional cost 2,000 2,800
Net savings 4,500 6,000
Less: Depreciation 2,125 3,440
PBT 2,375 2,560
Less: tax ---- ----
PAT 2,375 2,560
Add: depreciation 2,125 3,440
PATBD (inflow) 4,500 6,000
Calculation of depreciation: 9,000 − 500 18,000 − 800
cost − SV 4 5
Dept=
Life = 2,125 = 3, 440
Payback period
Initial investment 9,000
=2 yrs
18,000
= 3 yrs
PBP = 4,500 6 ,000
Average cash inf lows

71 | P a g e
17. Rank the following projects using NPV and PBP:
Project Initial investment Annual cash flows Life in years
s
A 5,00,000 1,00,000 7 years
B 6,00,000 1,50,000 5 years
C 6,50,000 1,25,000 9 years
D 8,00,000 2,00,000 5 years
E 7,50,000 2,20,000 6 years

Solution is as follows:
As the given cash inflows are of only one year, annuity concept has to be followed:

Project
PBP =
II Rank NPV = PV of cash flows − I I Rank
s
AAI PV of cash flows = Inflows x PVIFA
5,00,000 NPV = [1,00,000 x 4,868] – 5,00,000
A PBP = = 5 years 3 NPV = -13,200 3
1,00,000
6,00 ,000 NPV = [1,50,000 x 3.791] – 6,00,000
B PBP = = 4 years 2 NPV = -31,350 4
1,50,000
6,50 ,000 NPV = [1,25,000 x 5.759] – 6,50,000
C PBP = = 5.20 years 4 NPV = 69,875 2
1,25,000
8,00 ,000 NPV = [2,00,000 x 3.791] – 8,00,000
D PBP = = 4 years 2 NPV = -41,800 5
2,00 ,000
7,50,000 NPV = [2,20,000 x 4.355] – 7,50,000
E PBP = = 3.41 years 1 NPV = 2,08,100 1
2,20,000

72 | P a g e
18. United Industries Ltd., has an investment budget of Rs. 100 lakhs for 2005-2006. It has short
listed two projects A and B after completing the market and technical appraisal. The
management wants to complete financial appraisals before making the investments. Further
particulars regarding the two projects are given below:
Particulars A B
Investment required 100 lakhs 90 lakhs
Average annual cash flows before Dept and Tax 28 lakhs 24 lakhs
Salvage value ----------- ---------
Estimated life (years) 10 years 10 years
The company follows straight line method of charging depreciation. Its tax rate is 50%. You are
required to calculate payback period and internal rate of return of the two projects.

Solution is as follows:
Cash inflows are of annuity concept
Calculation of cash inflows (PATBD)
Particulars Project A Project B
PBD & T 28,00,000 24,00,000
Less: Depreciation 10,00,000 9,00,000
PBT 18,00,000 15,00,000
Less: tax 9,00,000 7,50,000
PAT 9,00,000 7,50,000
Add: depreciation 10,00,000 9,00,000
Inflows (PATBD) 19,00,000 16,50,000
Project - A
Calculation of PBP
Initial investment 1,00 ,00,000
PBP = = = 5.26 years
AAI 19,00,000

Calculation of IRR involves four steps:


Step 1
PBP = 5.26 years
Step 2
By referring Table 4 (PVIFA) PBP of 5.26 years falls in between 12% and 14% in 4 th year row.
Now let us find PV at both the discount factors as follows:

Step 3
PV @ LDF = Inflows x (PVIFAn=10, r=12) = 19,00,000 x 5.650 = 1,07,35,000
PV @ HDF = Inflows x (PVIFAn=10, r=14) = 19,00,000 x 5.216 = 99,10,400
Step 4

73 | P a g e
PV @ LDF − II ]
IRR = LDF +
[[[ PV @ LDF − PV @ HDF ] ]
x [ HDF − LDF ]

1 , 07 ,35 ,000 − 1 , 00 , 00 ,000 ]


IRR = 12 +
[[[
1 , 07 ,35 ,000 − 99 , 10 , 400 ] ]
x [ 14 − 12 ]

7 , 35 , 000
IRR = 12 + [
8 , 24 ,600
x2 ]
IRR = 12 + 1 .783 = 13.783 %
Project - B
Calculation of PBP
Initial investment 90 ,00 ,000
PBP = = = 5.455 years
AAI 16 ,50 ,000

Calculation of IRR involves four steps:


Step 1
PBP = 5.455 years
Step 2
By referring Table 4 (PVIFA) PBP of 5.26 years falls in between 12% and 14% in 4 th year row.
Now let us find PV at both the discount factors as follows:

Step 3
PV @ LDF = Inflows x (PVIFAn=10, r=12) = 16,50,000 x 5.650 = 93,22,500
PV @ HDF = Inflows x (PVIFAn=10, r=14) = 16,50,000 x 5.216 = 88,06,400
Step 4
[ PV @ LDF − II ]
IRR = LDF +
[
[ PV @ LDF − PV @ HDF ] ]
x [ HDF − LDF ]

93 , 22 ,500 − 90 ,00 ,000 ]


IRR = 12 +
[[[
93 , 22 ,500 − 88 , 06 , 400 ] ]
x [ 14 − 12 ]

3 , 22 ,500
IRR = 12 + [
5 , 16 , 100
x2 ]
IRR = 12 + 1 .250 = 13 .250 %

19. What is the internal rate of return on an investment which involves a current outlay of Rs.
3,00,000 and results in an annual cash inflows of Rs. 60,000 for 7 years?

Solution is as follows:
Calculation of IRR involves four steps:
Step 1
PBP = 3,00,000 / 60,000 = 5 years
Step 2
By referring Table 4 (PVIFA) PBP of 5 years falls in between 8% and 10% in 7 th year row. Now let
us find PV at both the discount factors as follows:
Step 3
PV @ LDF = Inflows x (PVIFAn=7, r=8) = 60,000 x 5.206 = 3,12,360
PV @ HDF = Inflows x (PVIFAn=7, r=10) = 60,000 x 4.868 = 2,92,080
Step 4

74 | P a g e
PV @ LDF − II ]
IRR = LDF +
[[[ PV @ LDF − PV @ HDF ] ]
x [ HDF − LDF ]

3 ,12 , 360 − 3 ,00 , 000 ]


IRR = 8 +
[[[
3 ,12 , 360 − 2, 92 , 080 ] ]
x [ 10 − 8 ]

12, 360
IRR = 8 + [
20 ,280
x2 ]
IRR = 8 + 1 . 219 = 9 .219 %
20. If an equipment costs Rs. 5,00,000 and lasts 8 years, what should be the minimum annual cash
inflow before it is worthwhile to purchase the equipment? Assume that the cost of capital is 10
per cent.

Solution is as follows:
Inflows = Initial Investment / PVIFAn=8, r=10
Inflows = 5,00,000 / 5.335 = 93,721

21. How much can be paid for a machine which brings in an annual cash inflows of Rs. 25,000 for 10
years. Assume that the discount rate is 12 per cent.

Solution is as follows:
Initial Investment = Inflows x PVIFAn=10,r=12%
Initial Investment = 25,000 x 5.650 = 1,41,250

22. The expected cash flows of a project are as follows:


Years 0 1 2 3 4 5
Cash flows 1,00,000 20,00 30,000 40,000 50,00 30,000
0 0
The cost of capital is 12 per cent. Calculate the following: Net present value, Benefit cost ratio,
IRR, PBP and ARR

Solution is as follows:
 NPV = Present value of cash inflows – initial investment
Calculation of Present value of cash inflows:

Years Cash inflows PV factor PV of cash Cumulative


inflows cash inflows
1 20,000 0.893 17,860 20,000
2 30,000 0.797 23,910 50,000
3 40,000 0.712 28,480 90,000
4 50,000 0.636 31,800 1,40,000
5 30,000 0.567 17,010 1,70,000
Total Present value of cash inflows 1,19,060
Less: Initial Investment 1,00,000
Net present value 19,060

 Benefit cost ratio (PI)

75 | P a g e
1,19,060
BCR = = .1.19
1,00,000

10,000
PBP = 3 + = 3 + 0.20 = 3.20 years
 50,000
Average inf lows 14,000
ARR = x 100 = x 100 = 28%
 Averge investment 50,000
o
Total inf lows ( PAT ) 70,000 (1, 70,000 − 1,00,000)
Average inf lows = = = 14 ,000
No.of years 5
Initial investment 1 , 00 ,000
Average investment = = = 50 ,000
o 2 2
 IRR
Calculation of IRR involves four steps:
Step 1
PBP = Initial Investment / Average cash inflows
PBP = 1,00,000 / 34,000 (1,70,000 / 5)= 2.941 years
Step 2
By referring Table 4 (PVIFA) PBP of 2.941 years falls in between 18% and 20% in 5 th year
row. Now let us find PV at both the discount factors as follows:
Year Cash inflows PV@20% PV@18% PV of PV of LDF
s HDF
1 20,000 0.833 0.847 16,660 16,940
2 30,000 0.694 0.718 20,820 21,540
3 40,000 0.579 0.609 23,160 24,360
4 50,000 0.482 0.516 24,100 25,800
5 30,000 0.402 0.437 12,060 13,110
Present values at HDF and LDF 96,800 1,01,750

Step 3
PV @ LDF = 1,01,750
PV @ HDF = 96,800
Step 4
[ PV @ LDF − II ]
IRR = LDF +
[
[ PV @ LDF − PV @ HDF ]
x [ HDF − LDF ]
]
[ 1 , 01 ,750 − 1 , 00 , 000 ]
IRR = 18 +
[ [ 1 , 01 ,750 − 96 ,800 ] ] x [ 18 − 20 ]

1 , 750
IRR = 18 + [ ]
4 , 950
x2
IRR = 18 + 0 . 707 = 18 .707 %

76 | P a g e
23. A company is considering an investment proposal to install a new milling control at a cost of Rs.
50,000. The facility has a life expectancy of 5 years without any salvage value. The firm uses SLM
of depreciation and the same is used for tax purposes. The tax rate is assumed to be 35%. The
estimated cash flows before depreciation and tax from the investment proposal are as follows:
Years 1 2 3 4 5
Cash flows 10,000 10,692 12,769 13,462 20,385
before tax
Calculate: payback period, ARR, NPV at 10 per cent and PI at 10% discount rate.

Solution is as follows:
Calculation of cash inflows (PATBD)
 PBP
Year PBD&T Dept PBT Tax PAT PATBD Cumulative
s 2 3 4(2-3) 5(4*t) 6 7(6+3) 8
1
1 10,000 10,00 ----- ----- ----- 10,000 10,000
2 10,692 0 692 242 450 10,450 20,450
3 12,769 10,00 2,769 969 1,800 11,800 32,250
4 13,462 0 3,462 1,212 2,250 12,250 44,500
5 20,385 10,00 10,385 3,635 6,750 16,750 61,250
0
10,00
0
10,00
0
11,25
0

5,500
PBP = 4 + = 4 + 0.33 = 4.33 years
16 ,750
Average income ( PAT ) 11,250/5 2,250
ARR = x 100 = x100= x 100 = 9%
 Average investment 50 ,000/2 25,000

Calculation of PV of cash inflows


Year PATBD PV@10% PV of cash flows
s
1 10,000 0.909 9,090

77 | P a g e
2 10,450 0.826 8,632
3 11,800 0.751 8,862
4 12,250 0.683 8,367
5 16,750 0.621 10,402
45,353

 NPV = PV of inflows – Initial investment = 45,353 – 50,000 = (-) 4,647

PV of inf lows 45,353


PI = = = 0.907
 Initial investment 50,000

24. Sulabh International is evaluating a project whose expected cash flows are as follows:
Year 0 1 2 3 4 5
Cash flows 10,00,000 1,00,000 2,00,00 3,00,000 6,00,000 3,00,000
0
a. What is the NPV of the project, if the discount rate is 14 per cent for the entire period?
b. What is the NPV of the project if the discount rate is 12 per cent for year 1 and rises
every year by 1 per cent?
Solution is as follows:
Calculation of PV of cash inflows

Years NPV at 14% discount rate NPV if discount rate


increased by 1% every year
Cash inflows PV@14 PV of PV PV cash flows
% cash flows factor
1 1,00,000 0.877 87,700 0.893 89,300
2 2,00,000 0.769 1,53,800 0.783 1,56,600
3 3,00,000 0.675 2,02,500 0.675 2,02,500
4 6,00,000 0.592 3,55,200 0.572 3,43,200
5 3,00,000 0.519 1,55,700 0.476 1,42,800
9,54,900 9,34,400

 NPV = PV of cash inflows – initial investment


NPV = 9,54,900 – 10,00,000 = (-) 45,100
 NPV = PV of cash inflows – initial investment
NPV = 9,34,400 – 10,00,000 = (-) 65,600

78 | P a g e
25. Your company is considering two mutually exclusive projects, A and B. project A involves an
outlay of RS. 100 million which will generate an expected cash inflow of Rs. 25 million per year
for 6 years. Project B calls for an outlay of Rs. 50 million which will produce an expected cash
inflow of Rs. 13 million per year for 6 years. The company cost of capital is 12 per cent.
Calculate the NPV and IRR of each project.

Solution is as follows:
In the given question cash flows are given only for one year, so it should be considered as
annuity value.
Project A
 NPV = [Inflows x PVIFAn=6, r=12%] – Initial investment
NPV = [25 x 4.111] – 100 = 103 – 100 = 3
 Calculation of IRR involves four steps:
Step 1
PBP = 100 / 25 = 4 years
Step 2
By referring Table 4 (PVIFA) PBP of 4 years falls exactly at 13%.
IRR is 13%

Project B
 NPV = [Inflows x PVIFAn=6, r=12%] – Initial investment
NPV = [13 x 4.111] – 50 = 53 – 50 = 3
 Calculation of IRR involves four steps:
Step 1
PBP = 50 / 13 = 3.846 years
Step 2
By referring Table 4 (PVIFA) PBP of 3.846 years falls between 14% and 15% in 6 th
year row. Determine the PV of cash inflows in step 3 taking annuity concept.
Step 3
PV @ LDF = 13 x 3.889 = 50.557

79 | P a g e
PV @ HDF = 13 x 3.784 = 49.192
Step 4

PV @ LDF − II ]
IRR = LDF +
[[[ PV @ LDF − PV @ HDF ] ]
x [ HDF − LDF ]

50. 557 − 50 ]
IRR = 14 +
[[[
50. 557 − 49. 192 ] ]
x [ 15 − 14 ]

0 .557
IRR = 14 + [ ]
1. 365
x1
IRR = 14 + 0 . 408 = 14. 408 %

26. The expected cash flows of a project are as follows:


Year 0 1 2 3 4 5
Cash flows 1,00,000 20,00 30,000 40,000 50,00 30,000
0 0
The cost of capital is 12 per cent. Calculate MIRR.

Solution is as follows:
TV
PVC =
[ 1 − MIRR ] n
TV = Terminal value (Future value of cash inflows)
PVC = Present value of cash outflow

Calculation of Terminal values


n−1 n−2 n−n
TV =P [ 1 + r ] + P [ 1 + r ] + −−−−−− P [ 1 + r ]
5−1 5−2 5−3 5−4 5−5
TV = 20,000 [1 + 0.14 ] + 30,000 [ 1 + 0.14 ] + 40,000 [1 + 0.14 ] + 50,000 [ 1 + 0.14 ] + 30,000 [ 1 + 0.14 ]
4 3 2 1 0
TV = 20,000 [1 .14 ] + 30,000 [ 1 .14 ] + 40,000 [ 1 .14 ] + 50,000 [ 1 .14 ] + 30,000 [ 1.14 ]
TV = 20,000 [1 .574 ] + 30,000 [1 .405 ] + 40,000 [ 1,.254 ] + 50,000 [ 1.120 ] + 30,000 [ 1 ]
TV = 31,480 + 42,150 + 50,160 + 56,000 + 30,000
TV = 2,09,790
PVC = 1,00,000

Calculation of MIRR

80 | P a g e
2,09,790
1,00,000 =
[ 1 − MIRR ]5
[ 1 − MIRR ]5 = 2,09,790
1,00,000
5
[ 1 − MIRR ] = 2.0979
1/5
1 − MIRR = [ 2.0979 ]
1 − MIRR = 1.160
MIRR = 1.160 − 1 = 0.16 or 16%

27. Your company is considering two projects M and N, each of which requires an initial outlay of
Rs. 50 million. The expected cash inflows from these projects are:
Year 1 2 3 4
Project M 11 million 19 million 32 million 37 million
Project N 38 million 22 million 18 million 10 million
a. What is the payback period for M and N?
b. If cost of capital is 14 per cent, what is the MIRR of each project?

Solution is as follows:
Calculation of present value of cash inflows using 14% as discount factor
Net Present Value
Project N Project N
Year
PV of PV of
s Inflows PV@14% Inflows PV@14%
cashflows cashflows
1 11 0.893 9.823 38 0.893 33.934
2 19 0.797 15.143 22 0.797 17.534
3 32 0.712 22.784 18 0.712 12.816
4 37 0.636 23.532 10 0.636 6.360
PV of cash inflows 71.282 PV of cash inflows 70.644
Less: initial investment 50.000 Less: cash outflow 50.000
NPV 21.282 NPV 20.644

MIRR
Project A

81 | P a g e
TV
PVC =
[ 1 − MIRR ] n
TV = Terminal value (Future value of cash inflows)
PVC = Present value of cash outflow
Calculation of Terminal values
TV =P [ 1 + r ]n−1 + P [ 1 + r ]n−2 + −−−−−− P [ 1 + r ] n−n
4−1 4−2 4−3 4−4
TV = 11 [ 1 + 0.14 ] + 19 [ 1 + 0.14 ] + 32 [ 1 + 0.14 ] + 37 [ 1 + 0.14 ]
3 2 1 0
TV = 11 [ 1 .14 ] + 19 [ 1 .14 ] + 32 [ 1 .14 ] + 37 [ 1 .14 ]
TV = 11 [ 1.482 ] + 19 [ 1.300 ] + 32 [ 1,.140 ] + 37 [ 1 ]
TV = 16.302 + 24.70 + 36.48 + 37
TV = 114 .482
PVC = 50
Calculation of MIRR
114. 482
150 =
[ 1 + MIRR ]4
[ 1 + MIRR ] 4 = 114 . 482
50
4
[ 1 + MIRR ] = 2.28964
1/4
1 + MIRR = [ 2 . 0979 ]
1 + MIRR = 1. 230
MIRR = 1 .230 − 1 = 0. 23 or 23 %
Project B
TV
PVC =
[ 1 − MIRR ] n
TV = Terminal value (Future value of cash inflows)
PVC = Present value of cash outflow
Calculation of Terminal values
TV =P [ 1 + r ]n−1 + P [ 1 + r ]n−2 + −−−−−− P [ 1 + r ] n−n
4−1 4−2 4−3 4−4
TV = 38 [ 1 + 0.14 ] + 22 [ 1 + 0.14 ] + 18 [ 1 + 0.14 ] + 10 [ 1 + 0.14 ]
3 2 1 0
TV = 38 [ 1 .14 ] + 22 [ 1 .14 ] + 18 [ 1 .14 ] + 10 [ 1 .14 ]
TV = 38 [ 1.482 ] + 22 [ 1.300 ] + 18 [ 1,.140 ] + 10 [ 1 ]
TV = 56.316 + 28.60 + 20.52 + 10
TV = 115 .436
PVC = 50
Calculation of MIRR

82 | P a g e
115.436
150 =
[ 1 + MIRR ]4
[ 1 + MIRR ] 4 = 115.426
50
4
[ 1 + MIRR ] = 2.30872
1/4
1 + MIRR = [ 2.30872 ]
1 + MIRR = 1.233
MIRR = 1.233 − 1 = 0.233 or 23.30%

28. A company is considering the following investment projects:


Projects Co C1 C2 C3
A - +10,000 ----- -----
B 10,000 +7,500 +7,500 -----
C - +2,000 +4,000 +12,000
D 10,000 +10,000 +3,000 +3,000
-
10,000
-
10,000
Rank the projects according to each of the following methods: PBP, ARR and NPV assuming 10%
as the discount rate.

Solution is as follows:
Pay Back Period:
 Project A: life is only one year

83 | P a g e
Initial investment 10,000
PBP = = = 1 year
AvergeCashflows 10,000
 Project B: life is two years. Cash flows are of annuity concept.
Initialinvestment 10,000
PBP = = = 1.33 years
AvergeCashflows 7,500
 Project C: life is three years. Cash flows are of single amount concept.
Amount required 4 ,000
PBP = MT + =2+ = 2 + 0.33 = 2.33 years
Amount available 12,000
 Project D: life is four years, but full amount of investment will be recovered by the end
of first year.
PBP = 1 year
Accounting Rate Return:
Here inflows are equal to Profit After Tax (PAT). Given are cash inflows (PATBD). To find the
income out of given cash inflows we need to subtract the amount of depreciation from the
given inflows.
In SLM Depreciation is equal to the cost of asset (initial investment)
Total Income = Total PATBD (inflows) – Depreciation (initial investment)
Project A = 10,000 – 10,000 = 0
Project B = 15,000 – 10,000 = 5,000
Project C = 18,000 – 10,000 = 8,000
Project D = 16,000 – 10,000 = 6,000

Average income = Total income / no of years:


Project A = 0
Project B = 5,000 / 2 = 2,500
Project C = 8,000 / 3 = 2,667
Project D = 6,000 / 3 = 2,000

Average investment = Initial Investment / 2


As investment is same for all the projects, average investment will be Rs. 5,000

 Project A
Average income 0
ARR = x 100 = x 100= 0%
Average investment 5,000
 Project B
Average income 2,500
ARR = x 100 = x 100= 50%
Average investment 5,000
 Project C
Average income 2,667
ARR = x 100 = x 100= 53.34%
Average investment 5,000
 Project D

84 | P a g e
Average income 2,000
ARR = x 100 = x 100= 40%
Average investment 5,000
Net Present Value
NPV = PV of cash inflow – Initial Investment
 Project A
NPV = 10,000 (0.909) – 10,000 = (-) 910
 Project B: annuity cash inflows
NPV = 7,500 (1.735) – 10,000 = 3,012.50
 Project C: single amount cash inflows
NPV = [2,000 (0.909) + 4,000 (0.826) + 12,000 (0.751)] – 10,000
NPV = [1,818 + 3,304 + 9,012] – 10,000 = 4,134
 Project D: single amount cash inflows
NPV = [10,000 (0.909) + 3,000 (0.826) + 3,000 (0.751) – 10,000
NPV = [9,090 + 2,478 + 2,253 = 13,821 – 10,000 = 3,821

Ranking of the project:

Project PBP Ranking ARR Rankin NPV Ranking


s g
A 1 yr 1 0% 4 (-) 910 4
B 1.33 yrs 2 50.00% 2 3,012.50 3
C 2.33 yrs 3 53.34% 1 4,134.00 1
D 1 yr 1 40.00% 3 3,821.00 2

29. Pentagon limited is evaluating a project that has the following cash flow stream associated with
it:
Years 0 1 2 3 4 5 6
Cash flows -120 - +20 +60 +8 +100 +120
80 0
Calculate MIRR if its cost of capital is 15%.

TV
PVC =
[ 1 − MIRR ] n
TV = Terminal value (Future value of cash inflows)

85 | P a g e
PVC = Present value of cash outflow
Calculation of Terminal values
TV =P [ 1 + r ]n−2 + P [ 1 + r ]n−3 + −−−−−− P [ 1 + r ]n−n
6−2 6−3 6−4 6−5 6−6
TV = 20 [ 1 + 0.15 ] + 60 [ 1 + 0.14 ] + 80 [ 1 + 0.14 ] + 100 [ 1 + 0.14 ] + 120 [ 1 ]
4 3 2 1
TV = 20 [ 1 .15 ] + 60 [ 1 .15 ] + 80 [ 1 .15 ] + 100 [ 1 .15 ] + 120 [ 1 ]
TV = 20 [ 1.749 ] + 60 [ 1.521 ] + 80 [ 1,.322 ] + 100 [ 1.150 ] + 120
TV = 34 .97 + 91.26 + 105.76 + 115 + 120
TV = 467
PVC = 120 + 80[0.0.870) = 189.60
Calculation of MIRR
467
189.60 =
[ 1 + MIRR ]6
[ 1 + MIRR ]4 = 467
189 .60
4
[ 1 + MIRR ] = 2.463
1/6
1 + MIRR = [ 2.30872 ]
1 + MIRR = 1.162
MIRR = 1.162 − 1 = 0.162 or 16.20%

30. OZS Enterprises is considering a project proposal for replacement on an old machine by new
machine. The old machine bought a few years ago has a book value of Rs. 4,00,000 and it can be
sold to realize a post tax salvage value of Rs. 5,00,000. It has a remaining life of five years after
which its net salvage value is expected to be Rs. 1,60,000. It is being depreciated annually at a
rate of 25 per cent under the WDV method. Working capital for the old machine is Rs. 4,00,000.
The new machine costs Rs. 16,00,000. it is expected to fetch a net salvage value of Rs. 8,00,000
after 5 years, when it will be no longer required. The depreciation rate applicable to it is 25 per
cent under the WDV method. The net working capital required for the machine is Rs. 5,00,000.
The new machine is expected to bring a saving of Rs. 3,00,000 annually in manufacturing costs.
Tax rate applicable to firm is 40 per cent. Given the above information advice the firm based on
incremental after tax cash flow.

86 | P a g e
Solution is as follows:
Working Note:
Calculation of depreciation as Written Down Value (MDV) method:
Inc. dept
Old Machine New Machine
Years 7 (5-3)
1 Opening Dept Closing Opening Dept Closing
value 2 3 value 3 value 4 5 value 6
1 4,00,000 1,00,000 3,00,000 16,00,00 4,00,000 12,00,000 3,00,000
2 3,00,000 75,000 2,25,000 0 3,00,000 9,00,000 2,25,000
3 2,25,000 56,250 1,68,750 12,00,00 2,25,000 6,75,000 1,68,750
4 1,68,750 42,188 1,26,562 0 1,68,750 5,06,250 1,26,562
5 1,26,562 31,641 94,921 9,00,000 1,26,563 3,76,687 94,921
6,75,000
5,06,250

Calculation of cash inflows for the replacement project


Particulars 0 1 2 3 4 5
Initial investment:
Cost of new asset 16,00,000
(+)Inc WC 1,00,000
(-)SV of old asset 5,00,000
Total Initial investment 12,00,000 ----- ----- ----- ----- -----
Operating cash flows:
Incremental Dept 3,00,000 2,25,000 1,68,750 1,26,562 94,921
Post tax savings Inc 1,20,000 90,000 67,500 50,625 37,968
Dept 3,00,000 3,00,000 3,00,000 3,00,000 3,00,000
Saving in Manf cost 1,80,000 1,80,000 1,80,000 1,80,000 1,80,000
Post tax savings in MC
Total savings ----- 3,00,00 2,70,00 2,47,500 2,30,62 2,17,968
0 0 5
Terminal cash flows:
SV of New fixed assets 8,00,000
(+)Recovery of WC 1,00,000
(-) SV of old assets 1,60,000
Total terminal cash flows ----- ----- ----- ----- ----- 7,40,000
Net cash flows 12,00,000 3,00,00 2,70,00 2,47,500 2,30,62 9,57,968
0 0 5

31. Teja International is determining the cash flow for a project involving replacement of an old
machine by a new machine. The old machine bought a few years ago has a book value of Rs.
8,00,000 and it can be sold to realize a post-tax salvage value of Rs. 9,00,000. It has a remaining
life of five years after which its net salvage value is expected to be Rs. 2,00,000. It is being
depreciated annually at a rate of 25 per cent under the WDV method. The working capital
associated with this machine is Rs.5,00,000.

87 | P a g e
The new machine costs Rs. 30,00,000. It is expected to fetch a new salvage value of Rs.
15,00,000 after five years. The depreciated rate applicable to it is 25 per cent under the WDV
method. The net working capital required for the new machine is Rs. 6,000,000 and is expected
to bring a saving of Rs. 6,50,000 annually in manufacturing costs. The tax rate applicable to the
firm is 30 per cent.
a. Estimate the cash flow associated with the replacement project.
b. What is the NPV of the replacement project if cost of capital is 14 per cent?
Solution is as follows:
Working Note:
Calculation of depreciation as Written Down Value (MDV) method:
Inc. dept
Old Machine New Machine
Years 7 (5-3)
1 Opening Dept Closing Opening Dept Closing
value 2 3 value 3 value 4 5 value 6
1 8,00,000 2,00,000 6,00,000 30,00,00 7,50,000 22,50,000 5,50,000
2 6,00,000 1,50,000 4,50,000 0 5,62,500 16,87,500 4,12,500
3 4,50,000 1,12,500 3,37,500 22,50,00 4,21,875 12,65,625 3,09,375
4 3,37,300 84,375 2,53,125 0 3,16,406 9,49,219 2,32,031
5 2,53,125 63,281 1,89,844 16,87,50 2,37,305 7,11,914 1,74,024
0
12,65,62
5
9,49,219

Calculation of cash inflows for the replacement project


Particulars 0 1 2 3 4 5
Initial investment:
Cost of new asset 30,00,000
(+)Inc WC 1,00,000
(-)SV of old asset 9,00,000
Total Initial investment 22,00,00 ----- ----- ----- ----- -----
0
Operating cash flows:
Incremental Dept 5,50,000 4,12,500 3,09,375 2,32,031 1,74,024
Post tax savings Inc 1,65,000 1,23,750 92,813 69,609 52,207
Dept 6,50,000 6,50,000 6,50,000 6,50,000 6,50,000
Saving in Manf cost 4,55,000 4,55,000 4,55,000 4,55,000 4,55,000
Post tax savings in MC
Total savings ----- 6,20,000 5,78,750 5,47,813 5,24,609 5,07,207
Terminal cash flows:
SV of New fixed assets 15,00,000
(+)Recovery of WC 1,00,000
(-) SV of old assets 2,00,000
Total terminal cash flows ----- ----- ----- ----- ----- 14,00,000
Net cash flows 22,00,00 6,20,000 5,78,750 5,47,813 5,24,609 19,07,207
PV@14% 0 0.877 0.769 0.675 0.592 0.519

88 | P a g e
PV of cash flows 22,00,00 5,43,740 4,45,059 3,69,774 3,10,569 9,89,840
0

32. Mahima enterprises considering replacing old machinery by a new machine. The old machine
bought few years ago has a book value of Rs. 90,000 and it can be sold for Rs. 25,000. It has a
remaining life of five years after which its net salvage value is expected to be Rs.10,000. It is
being depreciated annually at the rate of 20 per cent as per the WDV method.
The new machine costs Rs. 4,00,000. It is expected to fetch a net salvage value of Rs. 25,000
after 5 years. It will be depreciated annually at the rate of 25 per cent as per the WDV method.
Investment in working capital will not change with the new machine. The tax rate for the firm is
35 per cent. Estimate the cash flow associated with the replacement proposal, assuming other
costs remain unchanged and savings of Rs. 1,50,000.
Solution is as follows:
Working Note:
Calculation of depreciation as Written Down Value (MDV) method:
Inc. dept
Old Machine New Machine
Years 7 (5-3)
1 Opening Dept Closing Opening Dept Closing
value 2 3 value 3 value 4 5 value 6
1 90,000 18,000 72,000 4,00,000 80,000 3,20,000 62,000
2 72,000 14,400 57,600 3,20,000 64,000 2,56,000 49,600
3 57,600 11,520 46,080 2,56,000 51,200 2,04,800 39,680
4 46,080 9,216 36,864 2,04,800 40,960 1,63,840 31,744
5 36,864 7,373 29,491 1,63,840 32,768 1,31,072 25,395

Calculation of cash inflows for the replacement project


Particulars 0 1 2 3 4 5
Initial investment:
Cost of new asset 4,00,000
(+)Inc WC -----
(-)SV of old asset 25,000
Total Initial investment 3,75,00 ----- ----- ----- ----- -----
0
Operating cash flows:
Incremental Dept 62,600 49,600 39,680 31,744 25,395
Post tax savings Inc 21,910 17,360 13,888 11,110 8,888
Dept 1,50,000 1,50,000 1,50,000 1,50,000 1,50,000
Saving in Manf cost 97,500 97,500 97,500 97,500 97,500
Post tax savings in MC
Total savings ----- 1,19,41 1,14,86 1,11,38 1,08,61 1,06,388
0 0 8 0
Terminal cash flows:
SV of New fixed assets 25,000
(+)Recovery of WC -----
(-) SV of old assets 10,000
Total terminal cash flows ----- ----- ----- ----- ----- 15,000
Net cash flows 3,75,00 1,19,41 1,14,86 1,11,38 1,08,61 1,21,388

89 | P a g e
0 0 0 8 0

PROBLEMS & SOLUTIONS ON CAPITAL STRUCTURE

1. A Limited Company has a capital structure (all equity) comprising of Rs. 5,00,000, each
share of Rs. 10 each. The firm wants to raise an additional capital of s. 2,50,000 for
expansion programme. The firm has four alternative financial plans. If the firm is able to
earn an operating profit of Rs. 80,000 after additional investment and 50 per cent tax
rate. Calculate EPS for all four alternatives and select the preferable financial plan.
Financial plans are as follows:
a. Raise the entire amount by issue of new equity capital.
b. Raise 50% as equity capital and 50% as debt capital at 10% rate of interest.
c. Raise the entire amount as 12 per cent debentures
d. Raise 50% as equity capital and 50% as preference capital at 10% dividend.

Solution is as follows:
Working Note: calculation of No. of shares, Interest paid and preference dividend paid
for all the available financial plans: (old capital + new capital = total capital)
Plans No. of shares Interest paid Pref. dividend paid
equity capital = Debenture capital = preference capital
= x rate of interest x rate of PD
price per share
I 75,000 ----- -----
II 62,500 12,500 -----
III 50,000 30,000 -----
IV 62,500 ----- 12,500

Profitability statement
Financial Plans
Particulars
I II III IV
EBIT 80,000 80,000 80,000 80,000
Less: interest ----- 12,500 30,000 -----
PBT 80,000 67,500 50,000 80,000
Less: tax 40,000 33,750 25,000 40,000
PAT 40,000 33,750 25,000 40,000
Less: PD ----- ----- ----- 12,500
Earning to share 40,000 33,750 25,000 27,500
holders

90 | P a g e
earnings to SH 40,000 33,750 25,000 27,500
EPS=
No.of shares 75,000 62,500 50,000 62,500
= 0.53 =0.54 =0.50 =0.44

Financial plan II is the best alternative since its EPS is greater than other plans.

2. KPMG Ltd., has currently an ordinary share capital of Rs. 25,00,000, consisting of 25,000
shares of Rs. 100 each. The management is planning to raise another Rs. 20,00,000 to
finance a major programme of expansion through one of the following plans:
a. Entirely through ordinary shares
b. Rs. 10,00,000 through ordinary share and Rs. 10,00,000 through long-term
borrowings at 9% interest rate.
c. Rs. 5,00,000 through ordinary shares and Rs. 15,00,000 through long-term
borrowings at 9 per cent interest rate.
d. Rs. 10,00,000 through ordinary shares and Rs. 10,00,000 through preference
shares with 5 per cent preference dividend.
The company’s expected EBIT will be Rs. 8,00,000. Assuming a corporate tax rate
of 50 per cent, determine the EPS in each alternative and comment which
alternative is best and why?

Solution is as follows:
Working Note: calculation of No. of shares, Interest paid and preference dividend paid
for all the available financial plans: (old capital + new capital = total capital)
Plans No. of shares Interest paid Pref. dividend paid
equity capital = Debenture capital = preference capital
= x rate of interest x rate of PD
price per share
I 45,000 ----- -----
II 35,000 90,000 -----
III 30,000 1,35,000 -----
IV 35,000 ----- 50,000

Profitability statement
Financial Plans
Particulars
I II III IV
EBIT 8,00,000 8,00,000 8,00,000 8,00,000
Less: interest ----- 90,000 1,35,000 -----
PBT 8,00,000 7,10,000 6,65,000 8,00,000
Less: tax 4,00,000 3,55,000 3,32,500 4,00,000
PAT 4,00,000 3,55,000 3,32,500 4,00,000
Less: PD ----- ----- ----- 50,000
Earning to share 4,00,000 3,55,000 3,32,500 3,50,000
holders

91 | P a g e
earnings to SH 4 ,00 ,000 3,55,000 3,32,500 3,50,000
EPS=
No.of shares 45,000 35,000 30,000 35,000
= 8.88 =10.14 =11.08 =10.00

Financial plan III is the best alternative since its EPS is greater than other plans.

3. Penta Four Ltd., has an all equity capital structure consisting of 15,000 equity shares of
Rs. 100 each. The management is planning to raise another Rs. 25,00,000 to finance a
major expansion programme and is considering three alternative methods of financing:
a. To issue 25,000 equity shares of Rs. 100 each.
b. To issue 25,000, 8% debentures of Rs. 100 each.
c. To issue 25,000, 8% preference shares of Rs. 100 each.
The company’s expected EBIT will be Rs. 8,00,000. Assuming a corporate tax rate
of 50 per cent, determine the EPS in each financial plan and determine the best
one and why?

Solution is as follows:
Working Note: calculation of No. of shares, Interest paid and preference dividend paid
for all the available financial plans: (old capital + new capital = total capital)
Plans No. of shares Interest paid Pref. dividend paid
equity capital = Debenture capital = preference capital
= x rate of interest x rate of PD
price per share
I 40,000 ----- -----
II 15,000 2,00,000 -----
III 15,000 ----- 2,00,000

Profitability statement
Financial Plans
Particulars
I II III
EBIT 8,00,000 8,00,000 8,00,000
Less: interest ----- 2,00,000 -----
PBT 8,00,000 6,00,000 8,00,000
Less: tax 4,00,000 3,00,000 4,00,000
PAT 4,00,000 3,00,000 4,00,000
Less: PD ----- ----- 2,00,000
Earning to share 4,00,000 3,00,000 2,00,000
holders
earnings to SH 4 ,00 ,000 3,00,000 2,00,000
EPS=
No.of shares 40,000 15,000 15,000
= 10.00 =20.00 =13.33

92 | P a g e
4. A newly established company wishes to determine an appropriate capital structure. It
can issue 12 per cent debentures and 10 per cent preference share capital and the
existing tax rate is 35 per cent. The company requires Rs. 50,00,000 of capital. The
possible capital structure is as follows:
Plans 1 2 3 4 5
Debenture capital 00 30 3 50 50
0
Preference capital 00 00 2 00 20
0
Equity capital 100 70 5 50 30
0
If EBIT is 12 per cent, calculate EPS and suggest the best plan.

Solution is as follows:
Working Note: calculation of No. of shares, Interest paid and preference dividend paid
for all the available financial plans: (old capital + new capital = total capital)
Plans No. of shares Interest paid Pref. dividend paid
equity capital = Debenture capital = preference capital
= x rate of interest x rate of PD
price per share
I 50,000 ----- -----
II 35,000 1,80,000 -----
III 25,000 1,80,000 1,00,000
IV 25,000 3,00,000 -----
V 15,000 3,00,000 1,00,000

93 | P a g e
EBIT is 12% of capital = 50,00,000 x 12/100 = 6,00,000

Profitability statement
Financial Plans
Particulars
I II III IV V
EBIT 6,00,000 6,00,000 6,00,000 6,00,000 6,00,000
Less: interest ----- 1,80,000 1,80,000 3,00,000 3,00,000
PBT 6,00,000 4,20,000 4,20,000 3,00,000 3,00,000
Less: tax 2,10,000 1,47,000 1,47,000 1,05,000 1,05,000
PAT 3,90,000 2,73,000 2,73,000 1,95,000 1,95,000
Less: PD ----- ----- 1,00,000 ----- 1,00,000
Earning to share
3,90,000 2,73,000 1,73,000 1,95,000 95,000
holders
earnings to SH 3,90,000 2,73,000 1,73,000 1,95,000 95,000
EPS=
No.of shares 50,000 35,000 25,000 25,000 15,000
= 7.80 =7.80 =6.92 =7.80 =6.33

5. A manufacturing company has the following capital structure:


Particulars Amount
Equity shares (Rs. 50 each) 20,00,000
Retained earnings 10,00,000
10% Debentures 10,00,000
12% preference shares 10,00,000
Long-term debt at 11% 5,00,000
55,00,000
The present EBIT is Rs. 10,00,000. The company is contemplating an expansion
programme requiring an additional investment of Rs. 10,00,000.
It is hoped that the company will be able to maintain the same level of earnings. To raise
the additional capital the company has the following alternatives:
a. To issue debentures at 11 per cent.
b. To issue preference shares at 13 per cent.
c. To raise the entire capital through equity shares.
Examine these alternatives and suggest which alternative is best for the company.
Assume tax rate to be at 35 per cent.

Solution is as follows:
Working Note: calculation of No. of shares, Interest paid and preference dividend paid
for all the available financial plans: (old capital + new capital = total capital)
Plans No. of shares Interest paid Pref. dividend paid

94 | P a g e
equity capital = Debenture capital = preference capital
= x rate of interest x rate of PD
price per share
I 40,000 2,65,000 1,20,000
II 40,000 1,55,000 2,50,000
III 60,000 1,55,000 1,20,000

Profitability statement
Financial Plans
Particulars
I II III
EBIT 10,00,000 10,00,000 10,00,000
Less: interest 2,65,000 1,55,000 1,55,000
PBT 7,35,000 8,45,000 8,45,000
Less: tax 2,57,250 2,95,750 2,95,750
PAT 4,77,750 5,49,250 5,49,250
Less: PD 1,20,000 2,50,000 1,20,000
Earning to share 3,57,750 2,99,250 4,29,250
holders
earnings to SH 3,57 ,750 2,99,250 4 ,29,250
EPS=
No.of shares 40,000 40,000 60,000
= 8.94 =7.48 =7.15

6. A manufacturing company has the following capital structure:


Particulars Amount
Equity shares (Rs. 100 each) 20,00,000
Retained earnings 10,00,000
7% Debentures 8,00,000
9% preference shares 12,00,000
55,00,000
The company earns 12% on its capital. The company requires a sum of Rs. 25,00,000 to
finance its major expansion programme for which the following alternatives area
available to it:
a. Issue of 20,000 equity shares at a premium of Rs. 25 per share.
b. To issue preference shares at 10 per cent.
c. To raise through issue of 8% debentures..
Examine these alternatives and suggest which alternative is best for the company.
Assume tax rate to be at 50 per cent.
Solution is as follows:
Working Note: calculation of No. of shares, Interest paid and preference dividend paid
for all the available financial plans: (old capital + new capital = total capital)
Plans No. of shares Interest paid Pref. dividend paid
equity capital = Debenture capital = preference capital
= x rate of interest x rate of PD
price per share

95 | P a g e
I 40,000 56,000 1,08,000
II 20,000 56,000 3,58,000
III 20,000 2,56,000 1,08,000
EBIT is 12% of its total capital (55,00,000 + 25,00,000)
EBIT = 80,00,000 x 12% = 9,60,000
Profitability statement
Financial Plans
Particulars
I II III
EBIT 9,60,000 9,60,000 9,60,000
Less: interest 56,000 56,000 2,56,000
PBT 9,04,000 9,04,000 7,04,000
Less: tax 4,52,000 4,52,000 3,52,000
PAT 4,52,000 4,52,000 3,52,000
Less: PD 1,08,000 3,58,000 1,08,000
Earning to share 3,44,000 94,000 2,44,000
holders
earnings to SH 3,44,000 94,000 2,44,000
EPS=
No.of shares 40,000 20,000 20,000
= 8.60 =4.70 =12.20

7. A Company needs Rs. 10,00,000 for development of a new product. It would yield an
annual operating profit of Rs. 2,40,000, share price is Rs. 50 each. The company has the
objective of maximizing the earnings per share. Company is in the tax rate of 50 per
cent. funds can be raised at the following interest rates:
a. Upto Rs. 1,00,000 at 8 per cent
b. Over Rs. 1,00,000 to Rs. 5,00,000 at 12 per cent
c. Over Rs. 5,00,000 at 15 per cent.
The company has developed three following plans that are given below:
a. Raise Rs. 1,00,000 debt, with expected operating profit of Rs.3,40,000
b. Raise Rs. 3,00,000 debt, with expected operating profit of Rs. 4,40,000
c. Raise Rs. 6,00,000 dent, with expected operating profit of Rs. 5,90,000.
Calculate EPS for all the above financing plans and give the best plan based
on the higher EPS.

Solution is as follows:
Working Note:
a. Initially we should frame the capital structure of available plans

Plans Total Debt capital Equity capital


capital
A 10,00,000 1,00,000 9,00,000

96 | P a g e
B 10,00,000 3,00,000 7,00,000
C 10,00,000 6,00,000 4,00,000
b. Calculation of no. of shares and interest on debt capital for all the plans:
Plans No. of shares Interest on debt EBIT
A 18,000 8,000 3,40,000
B 14,000 8,000 + 24,000 = 32,000 4,40,000
C 8,000 8,000 + 48,000 + 15,000 = 71,000 5,90,000

Profitability statement
Financial Plans
Particulars
I II III
EBIT 3,40,000 4,40,000 5,90,000
Less: interest 8,000 32,000 71,000
PBT 3,32,000 4,08,000 5,19,000
Less: tax 1,66,000 2,04,000 2,59,500
Earning to share 1,66,000 2,04,000 2,59,500
holders
earnings to SH 1,66,000 2,04 ,000 2,59,500
EPS=
No.of shares 18,000 14,000 8,000
= 9.22 =14.57 =32.44

8. A Company has a share capital of Rs.2,00,000 divided in the shares of Rs. 10 each. It has
a major expansion programme requiring additional investment of Rs. 1,00,000. The
management is considering the following alternatives:
a. Issue of 10% debentures of Rs. 1,00,000.
b. Issue of 10,000, 15% preference shares of Rs. 10 each
c. Issue of 10,000 equity shares of Rs. 10 each.
The company present EBIT is Rs. 60,000 p.a. calculate the EPS for the above
three alternative financial plans presuming
i) EBIT continues to be the same, and
ii) EBIT Increase by Rs. 20,000. Tax rate is 50 per cent.

Solution is as follows:
Working Note: calculation of No. of shares, Interest paid and preference dividend paid
for all the available financial plans: (old capital + new capital = total capital)
Plans No. of shares Interest paid Pref. dividend paid
equity capital = Debenture capital = preference capital
= x rate of interest x rate of PD
price per share
I 20,000 10,000 -----
II 20,000 ----- 15,000
III 30,000 ----- -----

97 | P a g e
Profitability statement
Financial Plans Financial Plans
Particulars When EBIT = 60,000 When EBIT = 80,000
I II III I II III
EBIT 60,000 60,000 60,000 80,000 80,000 80,000
Less: interest 10,000 ----- ----- 10,000 ----- -----
PBT 50,000 60,000 60,000 70,000 80,000 80,000
Less: tax 25,000 30,000 30,000 35,000 40,000 40,000
PAT 25,000 30,000 30,000 35,000 40,000 40,000
Less: PD ----- 15,000 ----- ----- 15,000 -----
Earning to ESH 25,000 15,000 30,000 35,000 25000 40,000
earnings to SH 25,000 15,000 30,000 35,000 25,000 40,000
EPS=
No.of shares 20,000 20,000 30,000 20,000 20,000 30,000
= 1.25 =0.75 =1.00 = 1.75 =1.25 =1.33

9. AB Ltd. needs Rs. 10,00,000 for expansion. The expansion is expected to yield an annual
EBIT of Rs. 1,60,000. In choosing a financial plan, AB Ltd., has an objective of maximizing
earnings per share. It is considering the possibility of issuing equity shares and raising
debt of Rs. 1,00,000 or Rs. 4,00,000 or Rs. 6,00,000. The current market price per share
is Rs. 25 and is expected to drop to Rs. 20, if the bonds are borrowed in excess of Rs.
5,00,000. Funds can be borrowed at the rates indicated below:
a. Upto Rs. 1,00,000 at 8%
b. Over Rs. 1,00,000 and up to Rs. 5,00,000 at 12%
c. Over Rs. 6,00,000 at 18%.
Assume a tax rate of 50%. Determine the EPS for three financing alternatives and
suggest which plan is to be preferred.

Solution is as follows:
Working Note:
a. Initially we should frame the capital structure of available plans

Plans Total Debt capital Equity capital


capital
A 10,00,000 1,00,000 9,00,000
B 10,00,000 4,00,000 6,00,000
C 10,00,000 6,00,000 4,00,000

98 | P a g e
b. Calculation of no. of shares and interest on debt capital for all the plans:
Plans No. of shares Interest on debt
A 36,000 8,000
B 24,000 8,000 + 36,000 = 42,000
C 20,000 8,000 + 48,000 + 18,000 = 74,000

Profitability statement
Financial Plans
Particulars
I II III
EBIT 1,60,000 1,60,000 1,60,000
Less: interest 8,000 42,000 74,000
PBT 1,52,000 1,18,000 86,000
Less: tax 76,000 59,000 43,000
Earning to share 76,000 59,000 43,000
holders
earnings to SH 76,000 59,000 43,000
EPS=
No.of shares 36,000 24,000 20,000
= 2.11 =2.46 =2.15

PROBLEMS ON LEVERAGES

1. The operating and cost data of ABC Ltd., are:


a. Sales Rs. 20,00,000
b. Variable cost Rs. 14,00,000
c. Fixed costs Rs. 4,00,000 (including 15% interest on Rs. 10,00,000)
Calculate its operating, financial and combined leverage.

Solution is as follows:
Profitability statement
Particulars Amount
Sales 20,00,000
Less: variable 14,00,000
cost
Contribution 6,00,000
Less: fixed cost 2,50,000
EBIT 3,50,000
Less: interest 1,50,000
EBT 2,00,000

99 | P a g e
Calculation of leverages:
EBIT 3,50,000
Financial leverage = = = 1.75:1
EBT 2,00,000
Contribution 6,00,000
Operating leverage = = = 1.71:1
EBIT 3,50,000
Contribution 6,00,000
Combined leverage = = = 3:1
EBT 2,00,000

2. The data relating to two companies are as given below:


Particulars Company A Company B
Capital 6,00,000 3,50,000
12% Debentures 4,00,000 6,50,000
Output (units) per 60,000 15,000
annum 30 250
Selling price per unit 7,00,000 14,00,000
Fixed costs per annum 10 75
Variable cost per unit
You are required to calculate the operating leverage, financial leverage and combined
leverage of two companies.

Solution is as follows:
Profitability statement
Company A Company B
Particulars Rate/uni Amount Rate/unit Amount
t
Sales 30.00 18,00,000 250.00 37,50,000

100 | P a g e
Less: variable 10.00 6,00,000 75.00 11,25,000
cost
Contribution 20.00 12,00,000 175.00 26,25,000
Less: fixed cost 11.67 7,00,000 93.33 14,00,000
EBIT 8.33 5,00,000 81.67 12,25,000
Less: interest 48,000 78,000
EBT 4,52,000 11,47,000

Calculation of leverages:
EBIT 5,00 ,000
Financial leverage = = = 1.11:1
EBT 4,52,000
Contribution 12,00 ,000
Operating leverage = = = 2.40:1
EBIT 5,00,000
Contribution 12,00 ,000
Combined leverage = = = 2.65:1
EBT 4 ,52,000

3. The following is the income statement of XYZ Ltd., for the year 2010.
Particulars Amount
Sales 50.00
Less: variable cost 10.00
Contribution 40.00
Less: Fixed cost 20.00
EBIT 20.00
Less: Interest 05.00
EBT 15.00
Less: Tax at 40% 06.00
PAT 09.00
Less: Pref. dividend 01.00
Earnings available of ESH 08.00
Find out the degree of (a) operating leverage, (b) financial leverage and (c) combined
leverage.

101 | P a g e
Solution is as follows:
Calculation of leverages:
EBIT 20
Financial leverage = = = 1 .33:1
EBT 15
Contribution 40
Operating leverage = = = 2. 00 :1
EBIT 20
Contribution 40
Combined leverage = = = 2 . 67:1
EBT 15

4. The following is the balance sheet of Varun Ltd., as on 31-3-2008:


Liabilities Amount Assets Amount
Equity capital 1,80,000 Fixed assets 4,50,000
10% Debentures 2,40,000 Current assets 1,50,000
Retained Earnings 60,000
Current liability 1,20,000
Total 6,00,000 Total 6,00,000
The company’s total assets turnover ratio is 2.5 times. The fixed operating costs are Rs.
2,00,000. Variable operating cost ratio is 40 per cent. Income tax rate is 50 per cent.
Calculate the leverages.

Solution is as follows:
Initially we should calculate the amount of sales with the help of turnover:
Assets Turnover is 2.5 times. It means that sales (turnover) are 2.5 times of total
assets. So, sales amount is as follows:
Sales = 6,00,000 x 2.50 = 15,00,000

102 | P a g e
Profitability statement
Particulars Amount
Sales 15,00,000
Less: variable 6,00,000
cost
Contribution 9,00,000
Less: fixed cost 2,00,000
EBIT 7,00,000
Less: interest 24,000
EBT 6,76,000

Calculation of leverages:
EBIT 7 ,00,000
Financial leverage = = = 1.04:1
EBT 6 ,76,000
Contribution 9,00,000
Operating leverage = = = 1.29 :1
EBIT 7,00,000
Contribution 9,00,000
Combined leverage = = = 1.33: 1
EBT 6,76 ,000

5. ABC ltd., most recent balance sheet is as follows:


Liabilities Amount Assets Amount
Equity capital 60,000 Fixed assets 1,50,000
10% Debentures 80,000 Current assets 50,000
Retained Earnings 20,000
Current liability 40,000
Total 2,00,000 Total 2,00,000
The company’s total assets turnover is 3, its fixed operating costs are Rs. 1,00,000 and
its variable operating cost ratio is 40 per cent. the income tax rate is 50 per cent.
a. Calculate for the company, all the three types of leverages.
b. Determine the likely level of EBIT if the EPS is: Rs. 1, Rs. 3 and Nil.

Solution is as follows:
Initially we should calculate the amount of sales with the help of turnover:

103 | P a g e
Assets Turnover is 2.5 times. It means that sales (turnover) are 2.5 times of
total assets. So, sales amount is as follows:
Sales = 2,00,000 x 3 = 6,00,000

Profitability statement
Particulars Amount
Sales 6,00,000
Less: variable cost 2,40,000
Contribution 3,60,000
Less: fixed cost 1,00,000
EBIT 2,60,000
Less: interest 8,000
EBT 2,52,000

Calculation of leverages:
EBIT 2,60,000
Financial leverage = = = 1.03 :1
EBT 2,52,000
Contribution 3,60,000
Operating leverage = = = 1.39:1
EBIT 2,60,000
Contribution 3,60,000
Combined leverage = = = 1.43:1
EBT 2,52,000

6. The capital structure of XY Ltd., consists of the following securities:


10% Debentures Rs. 5,00,000
12% Preference Shares Rs. 1,00,000
Equity shares of Rs. 100 Rs. 4,00,000
Operating profit (EBIT) is Rs. 1,60,000 and the company is in 50% tax bracket.
a. Determine the company’s EPS
b. Determine the percentage change in EPS associated with 30% increase and
30% decrease in EBIT
c. Determine the degree of financial leverage
Solution is as follows:
Profitability statement
Particulars Amount
EBIT 1,60,000
Less: interest 50,000

104 | P a g e
EBT 2,10,000
Less: tax 1,05,000
PAT 1,05,000
Less: PD 12,000
Earnings to share 93,000
holders

Earnings to ESH 93,000


EPS = = = Rs. 23.25
a. No. of shares 4 ,000
b. EPS with 30% increase and decrease in EBIT
Profitability statement
Particulars Amount Amount
EBIT 2,08,000 1,12,000
Less: interest 50,000 50,000
EBT 1,58,000 62,000
Less: tax 79,000 31,000
PAT 79,000 31,000
Less: PD 12,000 12,000
Earnings to share 67,000 19,000
holders
earnings to SH 67,000 19,000
EPS=
No.of shares 4 ,000 4 ,000
= 16.75 =4.75

EBIT 2,08 ,000


Financial leverage = = = 1.31 :1
EBT 1,58 ,000
EBIT 1,12 ,000
Financial leverage = = = 1.81:1
c. EBT 62 ,000

7. The capital structure of Millennium software ltd., consists of an equity share capital of
Rs.10,00,000 (Rs. 100 each) and Rs. 10,00,000 of 10% debentures. Sales increased by
20% from 1,00,000 units to 1,20,000 units, the selling price is Rs. 10 per unit, variable
cost amount to Rs. 6 per unit and fixed expenses amount to Rs. 2,00,000. The IT rate is
assumed to be 50%.
a. You are required to calculate the following:
i. The percentage increase in EPS.
ii. The degree of financial leverage at 1,00,000 and 1,20,000 units.
iii. The degree of operating leverage at 1,00,000 and 1,20,000 units
b. Comment on the behavoiur of operating and financial leverage in relation to
increase in production from 1,00,000 to 1,20,000 units.

105 | P a g e
Solution is as follows:
Profitability Statement
Particulars Amount Amount
(1,00,000 units) (1,20,000 units)
Sales 10,00,000 12,00,000
Less: variable cost 6,00,000 7,20,000
Contribution 4,00,000 4,80,000
Less: fixed cost 2,00,000 2,00,000
EBIT 2,00,000 2,80,000
Less: interest 1,00,000 1,00,000
EBT 1,00,000 1,80,000
Less: tax 50,000 90,000
PAT 50,000 90,000
earnings to SH 50,000 90,000
EPS=
No.of shares 10,000 10,000
= 5.00 =9.00

4 [ 9−5 ]
Percentage of change in EPS = x 100 = 80 %
1. 5
EBIT 2,00,000
Financial leverage = = = 2:1
EBT 1,00,000
EBIT 2,80,000
Financial leverage = = = 1.56:1
2. EBT 1,80,000
Contribution 4,00,000
Operating leverage = = = 2:1
EBIT 2,00,000
Contribution 4,80,000
Operating leverage = = = 1.71:1
3. EBIT 2,80,000

8. The capital structure of Millennium software ltd., consists of an equity share capital of
Rs.10,00,000 (Rs. 10 each) and Rs. 10,00,000, 10% debentures. Sales increased by 25%
from 2,00,000 units to 2,50,000 units, the selling price is Rs. 10 per unit, variable cost
amount to Rs. 6 per unit and fixed expenses amount to Rs. 2,50,000. The IT rate is
assumed to be 50%.
a. You are required to calculate the following:
i. The percentage increase in EPS.
ii. The degree of financial leverage at 2,00,000 and 2,50,000 units.
iii. The degree of operating leverage at 2,00,000 and 2,50,000 units

Solution is as follows:
Profitability Statement

106 | P a g e
Particulars Amount Amount
(2,00,000 units) (2,50,000 units)
Sales 20,00,000 25,00,000
Less: variable cost 12,00,000 15,00,000
Contribution 8,00,000 10,00,000
Less: fixed cost 2,50,000 2,50,000
EBIT 5,50,000 7,50,000
Less: interest 1,00,000 1,00,000
EBT 4,50,000 6,50,000
Less: tax 2,25,000 3,25,000
PAT 2,25,000 3,25,000
earnings to SH 2,25,000 3,25,000
EPS=
No.of shares 1,00,000 1,00,000
= 2.25:1 =3.25:1

1 [ 3 . 25−2 . 25 ]
Percentage of change in EPS = x 100 = 44. 44 %
1. 2 .25
EBIT 5,50,000
Financial leverage = = = 1.22:1
EBT 4,50,000
EBIT 7 ,50,000
Financial leverage = = = 1.15:1
2. EBT 6 ,50,000
Contribution 8,00,000
Operating leverage = = = 1.45:1
EBIT 5,50,000
Contribution 10,00,000
Operating leverage = = = 1.33 :1
3. EBIT 7,50,000

9. Calculate operating and financial leverage under situations A and B of financial plan I
and II respectively. The following data is available:
Installed capacity 2,000 units: Actual production and sales 80 per cent; selling
price per unit Rs. 40; variable cost per unit is Rs. 30; fixed expenses: situation A –
Rs. 1,000; situation B – Rs. 3,000.
Capital structure is as follows:
Financial plans
Particulars
I II
Equity share capital 10,000 14,000
Debt (20%) 10,000 14,000

Solution is as follows:

107 | P a g e
Profitability Statement
Situation A Situation B
particulars Fixed cost Rs.1,000 Fixed cost Rs. 3,000
Plan 1 Plan 2 Plan 1 Plan 2
Sales (units) 1,600 1,600 1,600 1,600
Sales 64,000 64,000 64,000 64,000
Less: variable cost 48,000 48,000 48,000 48,000

Contribution 16,000 16,000 16,000 16,000


Less: fixed cost 1,000 1,000 3,000 3,000

EBIT 15,000 15,000 13,000 13,000


Less: interest 2,000 2,800 2,000 2,800
EBT 13,000 12,200 11,000 10,200
16,000/15,000 16,000 / 13,000
OL = C / EBIT
= 1.07:1 = 1.23
15000 15000 13000 13000
FL = EBIT / EBT 13000 12200 1,1000 10200
=1.15 =1.23 =1.18 =1.27

10. Calculate operating leverage and financial leverage under situations A, B and C and
financial plans 1, 2 and 3 respectively from the following information relating to the
operating and capital structure of XYZ Ltd., also find out the combinations of operating
and financial leverage which give the financial leverage which give the highest value and
the least value.
a. Installed capacity (units) 1,200
b. Actual production and sales (units) 800
c. Selling price per unit (Rs.) 15
d. Variable cost per unit (Rs.) 10
e. Fixed costs (Rs.)
i. Situation A 1,000
ii. Situation B 2,000

108 | P a g e
iii. Situation C 3,000
Capital structure
Financial plans
Particulars
1 2 3
Equity 5,000 7,500 2,500
Debt 5,000 2,500 7,500
Cost of debt 12% 12% 12%

Solution
Profitability Statement
Situation A Situation A Situation A
Particulars (fixed cost Rs. 1,000) (fixed cost Rs. 2,000) (Fixed cost Rs. 3,000)
Plan 1 Plan 2 Plan 3 Plan 1 Plan 2 Plan 3 Plan 1 Plan 2 Plan 3
Sales (units) 800 800 800 800 800 800 800 800 800
Sales 12,000 12,000 12,000 12,000 12,000 12,000 12,000 12,000 12,000
Less: variable cost 8,000 8,000 8,000 8,000 8,000 8,000 8,000 8,000 8,000

Contribution 4,000 4,000 4,000 4,000 4,000 4,000 4,000 4,000 4,000
Less: fixed cost 1,000 1,000 1,000 2,000 2,000 2,000 3,000 3,000 3,000

EBIT 3,000 3,000 3,000 2,000 2,000 2,000 1,000 1,000 1,000
Less: interest 600 300 900 600 300 900 600 300 900
EBT 2,400 2,700 2,100 1,400 1,700 1,100 400 700 100
4,000 / 3,000 4,000 / 2,000 4,000 / 1,000
OL= C / EBIT
= 1.33:1 = 2:1 = 4:1
3000 3000 3000 2000 2000 2000 1000 1000 1000
FL = EBIT / EBT 2,400 2,700 2,100 1,400 1700 1100 400 700 100
=1.25 =1.11 =1.43 =1.43 =1.18 =1.82 =2.25 =1.43 =10

11. A firm’s sales, variable cost and fixed cost amount to Rs. 75,00,000, Rs. 42,00,000 and
Rs. 6,00,000 respectively. It has borrowed Rs. 45,00,000 at 9 per cent and its equity
capitals total Rs. 55,00,000.
a. What is the firm’s ROI?
b. Does it have favourable financial leverage?
c. If the firm belongs to an industry whose assets turnover is 3 times, does it have
high or low asset leverage?
d. What are the operating, financial and combined leverages of the firm?
e. If the sales drop to Rs. 50,00,000, what will the new EBIT be?
f. At what level will the EBT of the firm equal to zero?

Solution is as follows:
1. ROI = EBIT / Investment *100

109 | P a g e
EBIT = Sales – VC – FC
EBIT = 75,00,000 – 42,00,000 – 6,00 000 = 27,00,000
Investment = Equity + Debt
Investment = 55,00,000 + 45,00,000 = 1,00,00,000
ROI = 27,00,000 / 1,00,00,000 *100 = 27%
2. Yes, the firm has financial leverage as its ROI is higher than rate of interest.
3. Assets turnover ratio = Sales /Total assets (total investment)
Assets turnover ratio = 75,00,000 / 1,00,00,000 = 0.75 times. Its turnover
ratio is lower than industry average of 3 times.
4. Leverages:
contribution ( sales − VC) 33 ,00 , 000
Operating leverage = = = 1.22:1
EBIT 27 ,00 , 000
EBIT 27 , 00, 000
Financial leverage = = = 1.18 :1
EBT ( EBIT −I ) 22 , 95, 000 (27 ,00 , 000 − 4 ,05 ,000 )
Contribution 33, 00 ,000
Combined leveragr = = = 1. 44 :1
EBT 22, 95 ,000

5. EBIT at sales level of Rs. 50,00,000:


Particulars Amount
Sales 50,00,000
Less: variable cost (56% of 28,00,000
sales)
Contribution 22,00,000
Less: fixed cost 6,00,000
EBIT 16,00,000
6. Zero EBT implies sales equal to cost (BEP)
BEP = FC / PV ratio
PV ratio = C / Sales *100 = 33,00,000 / 75,00,000 *100 = 44%
BEP = 10,05,000 (6,00,000 + 4,05,000) / 44% = Rs. 22,84,091

12. The selected financial data for A, B and C companies for the current year ended March 31 st are as follows:
Particulars A B C
Variable expenses as a percentage of sales 66.67 75 50
Interest expenses 200 300 1,000
Degree of operating leverage 5 6 6
Degree of financial leverage 3 4 2
Income-tax rate 0.35 0.35 0.35
Prepare income statement for A, B and C companies. Comment on the financial position and structure of
these companies.
Solution is as follows:
Working note:
a. Calculation of EBIT by using FL
Company A Company B Company C

110 | P a g e
EBIT EBIT EBIT
FL = FL = FL =
EBIT − I ( EBT ) EBIT − I ( EBT ) EBIT − I ( EBT )
X X X
3= 4= 2=
X − 200 X − 300 X − 1000
3 X − 600 = X 4 X −1200 = X 2 X − 2000 = X
2 X = 600 3 X = 1200 X = 2000
X ( EBIT )= 300 X ( EBIT )= 400 X ( EBIT )= 2000
Calculation of Contribution by using Operating Leverage
Contribution Contribution OL =
Contribution
OL = OL =
EBIT EBIT EBIT
X X X
5= 6= 6=
300 400 2000
X = 1 ,500 X = 2400 X = 12000
contribution = 1 ,500 contribution = 2400 contribution = 12000
Calculation of sales and VC by using contribution
Sales = VC – Contribution Sales = VC – Contribution Sales = VC – Contribution
VC is 66.67% of sales, so balance VC is 75% of sales, so balance VC is 50% of sales, so balance
33.33% of sales is contribution. 25% of sales is contribution. 50% of sales is contribution.
Contribution = 1500 (33.33% of Contribution = 2400 (25% of Contribution = 12000 (50% of
sales) sales) sales)
100% sales = 1500/33.33% 100% sales = 2400/25% 100% sales = 12000/50%
Sales = 4500 Sales = 9,600 Sales = 24000
VC = S – C VC = S – C VC = S – C
VC = 4500 – 1500 = 3000 VC = 9600 – 2400 = 7200 VC = 24000 – 12000 = 12000
Profitability Statement
Particulars Company A Company B Company C
Sales 4,500 9,600 24,000
Less: variable cost 3,000 7,200 12,000
Contribution 1,500 2,400 12,000
Less: fixed cost (b/f) 1,200 2,000 10,000
EBIT 300 400 2,000
Less: interest 200 300 1,000
EBT 100 100 1,000
Less: tax (50%) 35 35 350
PAT 65 65 650

SOLUTIONS OF WORKING CAPITAL MANAGEMENT

Gross Operating Cycle (GOC): the firm’s gross operating cycle can be determined as inventory
conversion period plus debtors’ conversion period.

GOC = ICP + DCP


GOC- Gross Operating Cycle
ICP – Inventory conversion period
DCP – Debtors’ conversion period

NOC (CCC)= GOC – APP


111 | P a g e
NOC - Net Operating Cycle or Cash Conversion Cycle (CCC)
GOC – Gross Operating Cycle
APP – Accounts Payable Period

1. ABC Company provided the following information and requested you to compute
operating cycle:
a. Sales: Rs. 3,00,000
b. Inventory: Opening Rs. 61,000; Closing Rs. 47,500
c. Receivables: Opening Rs.91,500; Closing Rs. 97,500
d. Cost of goods sold: 2,67,500

Solution is as follows:
OC = ICP + ARP
Average inventory
ICP =
cost of goods sold / no. of days
(61,000 + 47 ,500) /2 54,250
ICP = = = 74 days
2,67,500 / 365 733
Average debtors (91,500 + 97,500) / 2 94,500
ARP = = = = 115 days
Sales / no. of days 3,00,000 / 365 822
OC = 74 days + 115 days = 189 days

2. From the following information, calculate Cash Conversion Cycle.


a. Average use of inventories 80 days;
b. Accounts receivables collection period 50 days;
c. Accounts payable period 40 days

Solution is as follows:
CCC = OC − APP
CCC = (80 + 50) − 40 = 90 days

3. From the following financial data of Vamshadhara Co. Ltd., determine CCC.
a. Sales: Rs. 1,587.95 lakhs
b. Cost of goods sold: Rs. 1,406.27 lakhs
c. Inventory: Opening Rs. 195.82 lakhs; Closing Rs. 449.46 lakhs
d. Accounts receivables: Opening Rs. 423.03 lakhs; Closing Rs. 449.46 lakhs
e. Accounts payable: Opening: Rs. 140.40 lakhs; Closing Rs. 168.33 lakhs

Solution is as follows:
CCC = OC – APP

112 | P a g e
OC = ICP + ARP
Average inventory (195.82 + 202.29) /2 199.06
ICP = = = = 52 days
cost of goods sold / no. of days 1,406.27 / 365 3.85
Average debtors (423.03 + 449.46,) / 2 436.25
ARP = = = = 100 days
Sales / no. of days 1,587.95 / 365 4.35
OC = 52 days + 100 days = 152 days
Average creditors (140.40 + 168.33) / 2 154.37
APP = = = = 40 days
Cost of goods sold / noof days 1,406.27 / 365 3.85
CCC = 152 days − 40 days = 112 days

4. CP Paper Mills Ltd., has provided the following information and requested you to
calculate CCC.
a. Sales: Rs. 3,069.14 lakhs
b. Cost of Goods Sold: Rs. 2,686.24 lakhs
c. Inventory: Opening Rs. 615.61 lakhs; closing Rs. 482.79 lakhs
d. Receivables: Opening Rs. 926.30 lakhs; closing Rs. 986.58 lakhs
e. Payables: Opening Rs. 832.06 lakhs; Closing Rs. 901.79 lakhs

Solution is as follows:
CCC = OC – APP
OC = ICP + ARP
Average inventory (615.61 + 482 .79) /2 549.20
ICP = = = = 75 days
cost of goods sold / no. of days 2,686.24 / 365 7.36
Average debtors (926.30 + 986.58,) / 2 956.44
ARP = = = = 114 days
Sales / no. of days 3,069.14 / 365 8. 41
OC = 75 days + 114 days = 189 days
Average creditors (832.06 + 901.79) / 2 866 .93
APP = = = = 118 days
Cost of goods sold / noof days 2,686.24 / 365 7.36
CCC = 189 days − 118 days = 71 days

Working capital = Current assets – Current liabilities


Current assets = Inventory + Debtors + Cash
Inventory = Stock of Raw materials + Work-in-progress + finished goods
 Stock of raw materials consists of only one element of cost, i.e., raw materials.
Degree of completion will be as follows:
o Raw materials – 100% completion stage
 Stock of work-in-progress consists of all the three elements of cost, I.e., raw
materials, labour and overheads.
Degree of completion will be as follows:

113 | P a g e
o Raw materials – 100% completion stage
o Labour – less than 100% completion stage
o Overheads – less than 100% completion stage
 Stock of finished goods also consists of all the three element of cost.
Degree of completion stage will be as follows:
o Raw materials – 100% completion stage
o Labour – 100% completion stage
o Overheads – 100% completion stage

5. From the following information of VSGR Company Ltd., estimate the working capital
needed to finance a level of activity of 1,10,000 units of production after adding a 10 per
cent safety contingency:
Raw materials Rs. 78 per unit
Direct labour Rs. 29 per unit
Overheads (excluding depreciation) Rs. 58 per unit
Total cost Rs. 165 per unit
Profit Rs. 24 per unit

114 | P a g e
Selling Price Rs. 189 per unit
Additional Information:
 Average raw materials in stock: One month
 Average materials-in-progress (50% completion stage): Half a month
 Average finished goods in stock: One month
 Credit allowed by suppliers: One month
 Credit allowed to customers: Two months
 Time lag in payment of wages: One-and-half week
 overhead expenses: One month
One fourth of the sales are on cash basis. Cash balance is expected to be Rs.
2,15,000. You may assume that production is carried on evenly throughout the
year and wages and overhead expenses accrue similarly.

Solution is as follows:
Working Capital Requirement
Particulars Amount Amount
I – Current assets
Inventory
 stock of RM (1,10,000 x 78 x 1/12) 7,15,000
 stock of WIP: RM (1,10,000 x 78 x 0.5/12) 3,57,500
WIP (1,10,000 x 14.50 x 66,458
0.5/12) 1,32,916
FG (1,10,000 x 29 x 0.5/12) 15,12,50
 stock of FG: (1,10,000 x 165 x 1/12) 0
Debtors (82,500 x 165 x 2/12) 22,68,75 52,68,124
Cash 0
2,15,000
Total Current Assets 52,68,124
II – Current Liabilities
 creditors (1,10,000 x 78 x 1/12) 7,15,000
 outstanding expenses:
Wages (1,10,000 x 29 x 1.5/52) 92,019
Overheads (1,10,000 x 58 x 1/12) 5,31,667 13,38,686
Total Current Liabilities 13,38,686
Working capital 39,29,438
Add: 10% contingencies 3,92,944
Working capital required 43,22,382
6. XY Ltd is desirous to purchase a business and has consulted you, and one point on which
you are asked to advise them, is the average amount of working capital which will be
required in the first year’s working. You are given the following estimates and are
instructed to add 10 per cent to your computed figure to allow for contingencies.
Particulars Amount
Average amount backed up for stocks

115 | P a g e
 Stock of finished product 5,000
 Stock of stores and materials 8,000
Average credit given
 Inland sales, 6 weeks’ credit 3,12,000
 Export sales, 1.5 weeks’ credit 78,000
Average time lag in payment of wages and other outgoings
 Wages, 1.5 weeks 2,60,000
 Stock and materials, 1.5 months 48,000
 Rent and royalties, 6 months 10,000
 Clerical staff, 0.5 month 62,400
 Manager, 0.5 month 4,800
 Miscellaneous expenses, 1.5 months 48,000
Payment in advance:
 Sundry expenses (paid quarterly in advance) 8,000
Set up your calculation for the average amount of working capital required.

Solution is as follows:
Working Capital Requirement
Particulars Amount
I – Current assets
 Stock of finished goods 5,000
 Stock of stores and materials 8,000
 Debtors
Inland sales (3,12,000 x 6/52) 36,000
Export sales (78,000 x 1.5/52) 2,250
 Advance payment of sundry expenses (8,000 x ¼) 2,000
Total investment in Current Assets 53,250
II – Current Liabilities
 Wages (2,60,000 x 1.5/52) 7,500
 Stock/materials (48,000 x 1.5/12) 6,000
 Rent, royalties (10,000 x 6/12) 5,000
 Clerical staff (62,400 x 0.5/12) 2,600
 Manager (4,800 x 0.5/12) 200
 Miscellaneous expenses (48,000 x 1.5/12) 6,000
Total estimate of Current Liabilities 27,300
Net Working Capital (CA – CL) 25,950
Add: 10% contingency 2,595
Working Capital Requirement 28,545
7. While operating a project report on behalf of a client you have collected the following
facts. Estimate the net working capital required for that project. Add 10 per cent to your
computed figures to allow contingencies:
Particulars Amount
Estimated cost per unit cost of production
 Raw materials 80

116 | P a g e
 Direct labour 30
 Overheads (exclusive of depreciation, of Rs. 10/unit) 60
Total cash cost 170
Additional information:
 Selling price, Rs. 200 per unit
 Level of activity, 1,04,000 units of production per annum
 Raw materials in stock, average 4 weeks
 Work in progress (assume 50 per cent completion stage in respect of conversion
cost and 100 per cent in respect of materials) , average 2 weeks
 Finished goods in stock, average 4 weeks
 Credit allowed by suppliers, average 4 weeks
 Credit allowed to debtors, average 8 weeks
 Lag in payment of wages, average 1.5 weeks
 Cash at bank is expected to be, Rs. 25,000
You may assume that production is carried on evenly throughout the year (52
weeks) and wages and overhead occur similarly. All sales are on credit basis only.
Solution is as follows:
Working Capital Requirement
Particulars Amount Amount
I – Current assets
Inventory
 stock of RM (1,04,000 x 80 x 4/52) 6,40,000
 stock of WIP: RM (1,04,000 x 80x 2/52) 3,20,000
WIP (1,04,000 x 15 x 2/52) 60,000
FG (1,04,000 x 30 x 2/52) 1,20,000
 stock of FG: (1,04,000 x 170 x 4/52) 13,60,000
Debtors (1,04,000 x 170 x 8/52) 27,20,000
Cash 25,000
Total Current Assets 52,45,000 52,45,000
II – Current Liabilities
 creditors (1,04,000 x 80 x 4/52) 6,40,000
 outstanding expenses:
Wages (1,04,000 x 30 x 1.5/52) 90,000
Total Current Liabilities 7,30,000 7,30,000
Working capital 45,15,000
Add: 10% contingencies 4,51,500
Working capital required 49,66,500
8. From the following projections of XYZ Ltd for the next year, you are required to
determine the working capital required by the company:
 Annual sales, Rs. 14,40,000
 Cost of production (including depreciation of Rs. 1,20,000) Rs. 12,00,000
 Raw materials purchases, Rs. 7,05,000
 Monthly expenditure Rs. 30,000

117 | P a g e
 Estimated opening stock of raw materials Rs. 1,40,000
 Estimated closing stock of raw materials Rs. 1,25,000
 Inventory norms: raw materials 2 months; work-in-progress ½ month, and
finished goods 1 month
The firm enjoys a credit of half month on its purchases and allows one month
credit to its suppliers. On sales order the company receives ad advance of Rs.
15,000. You may assume that production is carried out evenly throughout the
year and minimum cash balance desired to be maintained is Rs. 35,000

Solution is as follows:
Working Capital Requirement
Particulars Amount Amount
I – Current assets
Inventory
 stock of RM:
Opening stock 1,40,00
Add: purchases 0
Less: closing stock 7,05,00
Annual consumption 0
Stock of RM (7,20,000 x 2/12) 1,25,00 1,20,000
 stock of WIP: (10,80,000 x 0.5/12) 0 45,000
 stock of FG: (10,80,000 x 1/12) 7,20,00 90,000
Debtors (10,80,000 x 1/12) 0 90,000
Cash 35,000
Total Current Assets 3,80,000
II – Current Liabilities
 creditors (7,05,000 x 0.5/12) 29,375
 Advance received from debtors 15,000
Total Current Liabilities 7,30,00 44,375
0
Working capital requirement 3,35,625

Cost of production is Rs. 10,80,000 (12,00,000 – 1,20,000)

9. A newly formed company has applied for a loan to commercial bank for financing its
working capital requirements. You are requested by the bank to prepare an estimate of
the requirements of the working capital for the company. Add 10% to your estimated

118 | P a g e
figure to cover unforeseen contingencies. The information about the profit and loss
account of the company is as under.
Particulars Amount
Sales 21,00,000
Less: cost of goods sold 15,30,000
Gross profit 5,70,000
Administrative expenses 1,40,000
Selling expenses 1,30,000
Profit before tax 3,00,000
Profit after tax 1,00,000

Cost of goods sold consists of the following:


Particulars Amount
Materials used 8,40,000
Wages & manufacturing expenses 6,25,000
Depreciation 2,35,000
Total 17,00,000
Less: stock of finished goods (10% not sold) 1,70,000
Cost of goods sold 15,30,000
The figures given above relate only to the goods that have been finished and not to
work-in-progress: goods equal to 15% of the year’s production (in terms of physical
units) are in progress on an average, requiring full materials but only 40% of other
expenses. The company believes in keeping two months consumption of material of
stock. Desired cash balance is Rs. 40,000.
Average time – log in payment of all expenses is 1 month; suppliers of materials
extended 1.5 months credit; sales are 20% cash; rest are at two months credit; 70% of
the income has to be paid in advance in quarterly instalments.
You can make such other assumptions as you deem necessary for estimating
working capital requirements.

Solution is as follows:
Working Capital Requirement
Particulars Amount Amount

119 | P a g e
I – Current assets
Inventory
 stock of RM (8,40,000 x 2/12) 1,40,000
 stock of WIP:
Raw Materials (8,40,000 x 15/100) 1,26,000
Labour and OH (6,25,000 x 40/100 x 15/100) 37,500
 stock of FG: (1,70,000 – 23,500*) 1,46,500
Debtors:
cost of goods sold 15,30,00
Less: depreciation (2,35,000 x 90/100) 0
Add: Administrative expenses 2,11,500
Add: selling expenses 1,40,000
Total 1,30,000
Credit sales (15,88,500 x 80/100 x 2/12) 15,88,50 2,11,800
Cash required 0 40,000
Total Current Assets 7,01,800
II – Current Liabilities
 Creditors (8,40,000 x 1.5/12) 1,05,000
 Outstanding expenses
Wages & manufacturing exp (6,25,000 x 52,083
1/12) 11.667
Administrative expenses (1,40,000 x 1/12) 10,833
Selling expenses (1,30,000x 1/12)
Total Current Liabilities 1,79,583
Working capital 5,22,217
Add: 10% contingency 52,222
Working capital requirement 5,74,439

Assumption:
1. Depreciation is not a cash expense and therefore, excluded from cost of
goods sold for the purpose of determining work-in-progress, finished and
investment in debtors

10. Heritage Garment ltd., is a famous manufacturer of garments. The finance manager of the
company is preparing its working capital forecast for the next year. After carefully screening all
the documents, he collected the following information.

120 | P a g e
Production during the previous year was 15,00,000 units. The same level of activity is
intended to be maintained during the current year. The expected ratios of cost to selling price
are:
Raw materials 40%
Direct wages 20%
overheads 20%
The raw materials ordinarily remain in stores for 3 months before production. Every unit
of production remains in the process for 2 months and is assumed to be consisting of 100% raw
material, 50% of wages and overheads. Finished goods remain in warehouse for 3 months.
Credit allowed by creditors is 4 months from the date of the delivery of raw materials and credit
given to debtors is 3 months from the date of dispatch.
The estimated cash balance to be held is Rs.2,00,000. Lag in payment of wages is half
month. Lag in payment of expenses is also half month. Selling price is Rs. 10 per unit. Both
production and sales are in a regular cycle. Make a provision of 10% for contingency (expect
cash)
Assuming you to be the finance manager for Heritage Garment Ltd., estimate the
working capital requirements of the company.

Solution is as follows:
Raw material is 40% of selling price = Rs. 4.00 per unit
Direct wages is 20% of selling price = Rs. 2.00 per unit
Overhears are 20% of selling price = Rs. 2.00 per unit
Working Capital Requirement
Particulars Amount Amount
I – Current assets
Inventory
 stock of RM (15,00,000 x 4 x 3/12) 15,00,000
 stock of WIP:
Raw Materials (15,00,000 x 4 x 2/12) 7,50,000
Direct wages (15,00,000 x 1 x 2/12) 2,50,000
Overheads (15,00,000 x 1 x 2/12) 2,50,000 12,50,000
 stock of FG: (15,00,000 x 8 x 3/12) 30,00,000
Debtors (15,00,000 x 8 x 3/12) 30,00,000
Cash required 2,00,000
Total Current Assets 89,50,000
II – Current Liabilities
 Creditors (15,00,000 x 4 x 4/12) 20,00,000
 Outstanding expenses
Wages (15,00,000 x 2 x 0.5/12) 1,25,000
Overheads (15,00,000 x 2 x 1/12) 2,50,000
Total Current Liabilities 23,75,000
Working capital 65,75,000
Add: 10% contingency 6,57,500
Working capital requirement 73,32,500
11. From the following data compute the estimated working capital of a company:
Raw materials Rs. 80 per unit
Direct labour Rs. 30 per unit

121 | P a g e
Overheads Rs. 50 per unit
Additional information:
 Selling price Rs. 200 per unit,
 Level activity 1,04,000 units of production per annum.
 Raw materials in stock average 4 weeks.
 Work in progress (50% completion stage in respect of conversion cost)
average 2 weeks.
 Finished goods in stock average 4 weeks.
 Credit allowed by suppliers – 4 weeks
 Credit allowed to debtors average 8 weeks
 Lag in payment of wages average 1.5 weeks
 Cash at bank is expected to be Rs. 25,000. You may assume that the
production is carried on evenly throughout the year (52 weeks) and wages
and overheads accrue similarly.
 All sales are on credit basis.

Solution is as follows:
Working Capital Requirement
Particulars Amount Amount
I – Current assets
Inventory
 stock of RM (1,04,000 x 80 x 4/52) 6,40,000
 stock of WIP: RM (1,04,000 x 80x 2/52) 3,20,000
DL (1,04,000 x 15 x 2/52) 60,000
OH (1,04,000 x 25 x 2/52) 1,00,000
 stock of FG: (1,04,000 x 160 x 4/52) 12,80,000
Debtors (1,04,000 x 160 x 8/52) 25,60,000
Cash 25,000
Total Current Assets 49,85,000 49,85,000
II – Current Liabilities
 creditors (1,04,000 x 80 x 4/52) 6,40,000
 outstanding expenses:
Wages (1,04,000 x 30 x 1.5/52) 90,000
Total Current Liabilities 7,30,000 7,30,000
Working capital 42,55,000
Add: 10% contingencies 4,25,500
Working capital required 46,80,500

12. A proforma cost sheet of RR & Co. Ltd., provides the following information. You are
required to estimate the working capital needed to finance a level of activity of 52,000
units of production:

122 | P a g e
Production Cost per unit
Raw materials 40
Direct labour 15
Overhead (excluding depreciation) 30
Total cost 85
Profit 30
Selling price 115
Additional information:
 Average raw materials in stock: one month
 Average materials in process (50% completion in conversion cost): half a
month
 Average finished goods in stock: one month
 Credit allowed by suppliers: one month
 Credit allowed to customers: two months
 Time lag in payment of wages: one and half weeks
 Time lag in payment of overhead expenses: one month
 Half the sale is on cash basis.
Cash balance is expected to be Rs. 12,500. You may assume that production is carried on
evenly throughout the year and wages and overhead expenses accrue similarly.

Solution is as follows:
Working Capital Requirement
Particulars Amount Amount
I – Current assets
Inventory
 stock of RM (52,000 x 40 x 1/12) 1,73,333
 stock of WIP: RM (52,000 x 40x 0.5/12) 86,667
DL (52,000 x 7.5 x 0.5/12) 16,250
OH (52,000 x 15 x 0.5/12) 32,500 1,35,417
 stock of FG: (52,000 x 85 x 1/12) 3,68,333
Debtors (26,000 x 85 x 2/12) 3,68,333
Cash 12,500
Total Current Assets 10,57,916
II – Current Liabilities
 creditors (52,000 x 40 x 1/12) 1,73,333
 outstanding expenses:
Wages (52,000 x 15 x 1.5/52) 22,500
Overheads (52,000 x 30 x 1/12) 1,30,00 1,52,500
0
Total Current Liabilities 3,25,833
Working capital requirement 7,32,083
13. Proforma cash sheet of a company provides the following particulars:
Material 40%

123 | P a g e
Direct labour 20%
Overheads 20%
The following information is also available:
 It is proposed to maintain a level of activity of 2,00,000 units
 Selling price is Rs. 12 per unit.
 Raw materials are expected to remain in store for an average period of one
month
 Materials will be in process on an average half a month at 50% completion.
 Finished goods are required to be in stock on average period of one month
 Credit allowed to debtors is two months
 Credit allowed by suppliers is one month
Estimate working capital required.

Solution is as follows:
Raw material is 40% of selling price = Rs. 4.80 per unit
Direct wages is 20% of selling price = Rs. 2.40 per unit
Overhears are 20% of selling price = Rs. 2.40 per unit
Working Capital Requirement
Particulars Amount Amount
I – Current assets
Inventory
 stock of RM (2,00,000 x 4.80 x 1/12) 80,000
 stock of WIP:
Raw Materials (2,00,000 x 4.80 x 40,000
0.5/12) 10,000
Direct wages (2,00,000 x 1.20 x 0.5/12) 10,000 60,000
Overheads (2,00,000 x 1.20 x 0.5/12) 1,60,000
 stock of FG: (2,00,000 x 9.60 x 1/12) 3,20,000
Debtors (2,00,000 x 9.60 x 2/12)
Total Current Assets 6,20,000
II – Current Liabilities
 Creditors (2,00,000 x 4.80 x 1/12) 80,000
Total Current Liabilities 80,000
Working capital 5,40,000

14. Foods Ltd., is presently operating at 60% level producing 36,000 packets of snacks foods
and proposed to increase the capacity utilization in the coming year by 33.33 per cent
over the existing level of production:

124 | P a g e
Raw material Rs. 40 per unit
Wages Rs. 20 per unit
Variable overheads Rs. 20 per unit
Fixed overhead Rs. 10 per unit
Profit Rs. 30 per unit
Selling price Rs. 120 per unit
 Raw materials will remain in stores for one month before being issued for
production. Material will remain in process for further one month. Suppliers
grant 3 months credit to the company.
 Finished goods remain in godown for one month
 Debtors are allowed credit for 2 months
 Lag in wages and overhead payment is one month
Prepare working capital requirement at the new level, assuming that a
minimum cash balance of Rs. 19,500 has to be maintained.

Solution is as follows:
Existing capacity utilization = 36,000
Proposed capacity utilization = 36,000 + 33.33% of 36,000 = 48,000 units

Working Capital Requirement


Particulars Amount Amount
I – Current assets
Inventory
 stock of RM (48,000 x 40 x 1/12) 1,60,000
 stock of WIP:
Raw Materials (48,000 x 40 x 1/12) 1,60,000
Direct wages (48,000 x 10 x 1/12) 40,000
Overheads (48,000 x 10 x 1/12) 40,000 2,40,000
 stock of FG: (48,000 x 80 x 1/12) 3,20,000
Debtors (48,000 x 80 x 2/12) 6,40,000
Cash balance 19,500
Total Current Assets 13,79,500
II – Current Liabilities
 Creditors (48,000 x 40 x 3/12) 4,80,000
 Outstanding expenses
Wages (48,000 x 20 x 1/12 ) 80,000
Overheads (48,000 x 20 x 1/12) 80,000 1,60,000
Total Current Liabilities 6,40,000
Working capital 7,39,500

125 | P a g e

Potrebbero piacerti anche